75

Click here to load reader

Document1

Embed Size (px)

Citation preview

Page 1: Document1

1. Which is the primary goal of community health nursing?

A. To support and supplement the efforts of the medical profession in the promotion of health and prevention ofB. To enhance the capacity of individuals, families and communities to cope with their health needs C. To increase the productivity of the people by providing them with services that will increase their level of healthD. To contribute to national development through promotion of family welfare, focusing particularly on mothers and children

2. CHN is a community-based practice. Which best explains this statement?

A. The service is provided in the natural environment of peopleB. The nurse has to conduct community diagnosis to determine nursing needs and problemsC. The service are based on the available resources within the communityD. Priority setting is based on the magnitude of the health problems identified

3. Population- focused nursing practice requires which of the following processes?

A. Community organizing .B. Nursing, processC. Community diagnosisD. Epidemiologic process

4. RA 1054 is also known as the Occupational Health Act. Aside from the number of employees, what other factor must be considered in determining the occupational health privileges to which the workers will be entitled?

A. Type of occupation,: agriculture, commercial, industrialB. Location of the workplace in relation to health facilitiesC. Classification of the business enterprise based on net profitD. Sex and age composition of employees

5. A business firm must employ an occupational health nurse when it has at least how many employees.

A. 21B. 101C. 201D. 301

6. When the occupational health nurse employs ergonomic principles, she is performing which of her roles?

A. Health care providerB. Health educatorC. Health care coordinatorD. Environment manager

7. A garment factory does not have an occupational nurse. Who shall provide the occupational health needs of the factory workers?

A. Occupational health nurse at the Provincial Health OfficeB. Physician employed by the factoryC. Public Health nurse of the RHU of their municipality D. Rural Sanitary inspector of the RHU in their municipality

8. "Public health services are given free of charge". Is this statement true or false?

A. The statement is true; it is the responsibility of government to provide haste servicesB. The statement is false; people pay indirectly for public health servicesC. The statement may be true or false; depending on the Specific service requiredD. The statement may be true or false; depending on policies of the government concerned.

9. According to C.E. Winslow, which of the following is the goal of Public Health?

A. For people to attain their birthrights and longevityB. For promotion of health and prevention and diseasesC. For people to have access to basic health servicesD. For people to be organized in their health efforts

10. We say that a Filipino has attained longevity when he is able to reach the average life span of Filipinos. What other statistic may be used to determine attainment of longevity?

A. Age-specific mortality rate B. Proportionate mortality rate C. Swaroop's index D. Case fatality rate

11. Which of the following is the most prominent feature of public health nursing?

A. It involves providing home care to sick people who are not confined in the hospitalB. Services are provided free of charge to people within the catchment area C. The public health nurse functions as part of a team providing a public health nursing serviceD. Public health nursing focuses on preventive, not curative services

12. According to Margaret Shetland, the philosophy of public health nursing is based on which of the following?

A. Health and longevity as birthrightsB. The mandate of the state to protect the birthrights of its citizens C. Public health nursing as a specialized field of nursing D. The worth and dignity of man

13. Which of the following is the mission of the Department of Health?

A. Health for all FilipinosB. Ensure the accessibility and quality of health C. Improve the general health status of the population D. Health in the hands of the Filipino people by the year 2020

14. Region IV Hospital is classified as what level of facility?

A. Primary B. SecondaryC. IntermediateD. Tertiary

15. What is true of primary facilities?

A. They are usually government-run B. Their services are provided on an out-patient basis C. They are training facilities for health professionals D. A community hospital is an example of this level of health facilities

Page 2: Document1

16. Which is an example of the school nurse's health care provider function?

A. Requesting for BCG from the RHU for school entrance immunizationB. Conducting random classroom inspection during measles epidemic C. Taking remedial action on an accident hazard in the school playground D. Observing places in the school where pupils spend their free times

17. When the nurse determines whether resources were maximized in implementing Ligtas Tigdas, she is evaluating:

A. Effectiveness B. Efficiency C. AdequacyD. Appropriateness

18. You are a new B.S.N. graduate. You want to become a Public Health Nurse. Where will you apply?

A. Department of HealthB. Provincial Health Office C. Regional Health OfficeD. Rural Health Unit

19. RA 7160 mandates devolution of basic services from the national government to local government units. Which of the following is the major goal of devolution?

A. To strengthen local government unitsB. To allow greater autonomy to local government units.C. To empower the people and promote their self-reliance D. To make basic services more accessible to the people

20. Who is the Chairman of the Municipal Health Board?

A. MayorB. Municipal Health Officer C. Public Health Nurse D. Any qualified physician

21. Which level of health facility is the usual point of entry of a client into the health care delivery system?

A. PrimaryB. Secondary

C. IntermediateD. Tertiary

22. The public health nurse is the supervisor of rural health midwives. Which of the following is a supervisory function of the pubic health nurse?

A. Referring cases or patients to the midwife B. Providing technical guidance to the midwifeC. Proving nursing care to cases referred by the midwife D. Formulating and implementing training programs for midwives

23. One of the participants in a hilot training class asked you to whom she should refer a patient in labor who develops a complication. You will answer, to the;

A. Public health nurseB. Rural health midwife C. Municipal health officer D. Any of these health professionals

24. You are the public health nurse in a municipality with a total population of about 20,000. There are3 health midwives among the RHU personnel. How many more midwife items will the RHU need?

A. 1B. 2C. 3D. 4

25. If the RHU needs additional midwife items, you will submit the request for additional midwife items for approval to the:

A. Rural Health Unit B. District Health Office C. Provincial Health Office D. Municipal Health Board

26. As an epidemiologist, the nurse is responsible for reporting cases or notifiable diseases. What law mandates reporting cases of notifiable diseases?

A. Act 3573B. RA.3753C. RA 1054 D. RA 1082

27. According to Freeman and Heinrich, community health nursing is a developmental service. Which of the following best illustrates this statement?

A. The community health nurse continuously develops himself personally and professionally B. Health education and community organizing are necessary in providing community health servicesC. Community health nursing in intended primarily for health promotion and prevention and treatment of disease. D. The goal of community health nursing is to provide nursing services to people in their own place of .residence

28. Which disease was declared through Presidential Proclamation No. 4 as a target for, eradication in the Philippines?

A. PioliomyelitisB. MeaslesC. RabiesD. Neonatal Tetanus

29. The public health nurse is responsible for presenting the municipal health statistics using graphs and tables. To compare the frequency of the leading causes of mortality in the municipality, which graph will you prepare?

A. LineB. BarC. PieD. Scatter diagram

30. Which step in community organizing involves training of potential leaders in the community?

A. IntegrationB. Community organizationC. Community studyD. Core group formation

31. In which step are plans formulated for solving community problems?

A. MobilizationB. Community organization C. Follow-up/extensionD. Core group formation

32. The public health nurse takes an active role in community participation. What is the primary goal of community

Page 3: Document1

organizing?

A. To educate the people regarding community health problems B. To mobilize the people to resolve community health problemsC. To maximize the community's resources in dealing with health problems

33. An indicator of success in community organizing is when people are able to:

A. Participate in community activities for the solution of a community problemB. Implement activities for the solution of the community problem C. Plan activities for the solution of the community problemD. Identify the health problem as a common concern

34. Tertiary prevention is needed in which stage of the natural history of disease?

A. Pre-pathogenesis B. PathogenesisC. PredromalD. Terminal

35. Isolation of a child with measles belongs to what level of prevention?

A. PrimaryB. SecondaryC. IntermediateD. Tertiary

36. On the other hand, Operation Timbang is_____ prevention?

A. PrimaryB. SecondaryC. IntermediateD. Tertiary

37. Which type of family-nurse contact will provide you with the best opportunity to observe family dynamics?

A. Clinic consultationB. Group conferencesC. Home visitD. Written communication

38. The typology of family nursing problems is used in the statement of nursing diagnosis in the care of families. The youngest child of the delos Reyes family has been diagnosed as mentally

retarded. This is classified as:

A. Health threat B. Health deficit C. Foreseeable crisisD. Stress point

39. The delos Reyes coupled have 6-year old child entering school for the first time. The delos Reyes family has a:

A. Health threat B. Health deficitC. Foreseeable crisisD. Stress point

40. Which of the following is an advantage of a home visit?

A. It allows the nurse to provide nursing care to a greater number of people B. It provides an opportunity to do first hand appraisal of the home situation C. It allows sharing of experience among people with similar health problems D. It develops the family's initiative in providing for health needs of its members

41. Which is CONTRARY to the principles in planning a home visit?

A. A home visit should have a purpose of objectiveB. The plan should revolve around the family health .needs C. A home visit should be conducted in the manner prescribed by RHU D. Planning of continuing care should involve a responsible-family member

42. The PHN bag is an important tool in providing nursing care during a home visit. The most important principle in bag technique states that it;

A. Should save time and effortB. Should minimize if not totally prevent the spread of infection C. Should not overshadow concern for the patient and his familyD. May be done in variety of ways depending on the home situation, etc.

43. To maintain the cleanliness of the bag and its contents, which of the following must the nurse do?

A. Wash his/her hands before and after providing nursing care to the family

membersB. In the care of family member's, as much as possible, use only articles taken from the bagC. Put on an apron to protect her uniform and fold it with the right side out before putting it back into the bag.D. At the end of the visit, fold the lining on which the bag was placed, ensuring that the contaminated side is on the outside.

44. The public health conducts a study on the factors contributing to the high morality rate due to heart diseases in the municipality where she works. Which branch of epidemiology does the nurse practice in this situation?

A. DescriptiveB. AnalyticalC. TherapeuticD. Evaluation

45. Which of the following is a function of epidemiology?

A. Identifying the disease condition based on manifestations presented by a client B. Determining factors that contributed to the occurrence of pneumonia in a 3 year oldC. Determining the efficacy of the antibiotic used in the treatment of the 3 year old client with pneumoniaD. Evaluating the effectiveness of the implementation of the Integrated Management of Childhood Illness

46. Which of the following is an epidemiologic function of the nurse during an epidemic?

A Conducting assessment of suspected cases to detect the communicable diseases

B. Monitoring the condition of the cases affected by the communicable disease C. Participating in the investigation to determine the source of epidemic D. Teaching the community on preventive measures against the disease

47. The primary purpose of conducting an epidemiologic investigation is to;

A. Delineate the etiology of the epidemicB. Encourage cooperation and support of the communityC. Identify groups who are at risk of

Page 4: Document1

contracting the diseaseD. Identify geographical location of cases of the disease in the community

48. Which is a characteristic of person-to-person propagated epidemic?

A. There are more cases of the disease than expected B. The disease must necessarily be transmitted through a vector C. The spread of the disease can be attributed to a common vehicle D. There is gradual build up of cases before we epidemic becomes easily noticeable

49. In the investigation of an epidemic, you compare the present frequency of the disease with the usual frequency at this time of the year in this community. This is done during which stage of the investigation?

A. Establishing the epidemicB. Testing the hypothesis C. Formulation of the hypothesis D. Appraisal of facts

50. The number of cases of Dengue fever usually increases towards the end of the rainy season. This pattern of occurrence of Dengue fever is best described as;

A. Epidemic occurrenceB. Cyclical variation C. Sporadic occurrence D. Secular occurrence

51. In the year 1980, the World Health Organization declared the Philippines, together with some other countries in the Western Pacific Region, “free" of which disease?

A. Pneumonic plaque B. Poliomyelitis C. Small pox D. Anthrax

52. In the census of the Philippines in 1995, there were about 35,299,000 males and about 34,968,000 females. What is the sex ratio?

A. 99.06:100B. 100.94:100C. 50.23%D. 49.76%

53. Primary health care is a total approach to community development. Which of the following is an indicator of success in the use of the primary health care approach?

A. Health services are provided free of charge to individuals and families B. Local officials are empowered as the major decision makers in matters of health

C. Health workers are able too provide care based on identified health needs of the peopleD. Health programs are sustained according to the level of development of the community

54. Sputum examination is the major screening tool for pulmonary tuberculosis. Clients would sometimes get false negative results in this exam. This means that the test is not perfect in terms of which characteristic of a diagnostic examination?

A. EffectivenessB. EfficacyC. SpecificityD. Sensitivity

55. Use of appropriate technology requires knowledge of indigenous technology. Which medical herb is given for fever, headache and cough?

A. SambongB. Tsaang gubatC. AkapulkoD. Lagundi

56. What law created the Philippine institute of Traditional and Alternative Health Care?

A. RA 8483B. RA4823C. RA 2483D. RA 3482

57. In traditional Chinese medicine, the yielding, negative and feminine force is termed:

A. YinB. YangC. QiD. Chai

58. What is the legal basis of Primary Health Care approach in the Philippines?

A. Alma Ata Declaration of PHCB. Letter of Instruction No 949C. Presidential Decree No. 147D. Presidential Decree 996

59. Which of the following demonstrates inter-sectoral linkages?

A. Two-way referral systemB. Team approachC. Endorsement done by a midwife to another midwifeD. Cooperation between PHN and public school teacher

60. The municipality assigned to you has a population of about 20/000. Estimate the number of 1-4 year old children who be given Retinol capsule 200.000 every 6 months.

A. 1,500B. 1,800C. 2,000D. 2,300

61. Estimate the number of pregnant women who will be given tetanus toxoid during an immunization outreach activity in a barangay with a population of about 1,500.

A. 265B. 300C. 375D. 400

62. To describe the sex composition of the population, which demographic tool may be used?

A. Sex ratio B. Sex proportion C. Population pyramid D. Any of these maybe used

63. Which of the following is a natality rate?

A. Crude birth rateB. Neonatal mortality rateC. Infant mortality rate D. General fertility rate

64. You are computing the crude rate of your municipality, with a total population

Page 5: Document1

o about 18,000 for last year. There were 94 deaths. Among those who died, 20 died because of diseases of the heart and 32 were aged 50 years or older. What is the crude death rate?

A. 4.1/1000B. 5.2/1000C. 6.3/1000D. 7.3/1000

65. Knowing that malnutrition is a frequent community health problem, you decided to conduct nutritional assessment. What population is particularly susceptible to protein energy malnutrition (PEM)?

A. Pregnant women and the elderlyB. Under 5 year old childrenC. 1-4 year old childrenD. School age children

66. Which statistic can give the most accurate reflection of the health status of a community?

A. 1-4 year old age-specific mortality rate B. Infant mortality rateC. Swaroop's index D. Crude death rate

67. In the past year, Barangay A had an average population of 1655. 46 babies were born in that year, 2 of whom died less than 4 weeks after they were born. They were 4 recorded stillbirths. What is the neonatal mortality rate?

A. 27.8/1000B. 43.5/1000C. 86.9/1000D. 130.4/1000

68. Which statistic best reflects the nutritional status of a population?

A. 1-4 year old age-specific mortality rate B. Proportionate mortality rateC. Infant mortality rateD. Swaroop's index

69. What numerator is used in computing general fertility rate?

A. Estimated midyear population B. Number of registered live births C. Number of pregnancies in the year D. Number of females of reproductive age

70. You will gather data for nutritional assessment of a purok. You will gather information only from families with members who belong to the target population for PEM. What method of delta gathering is best for this purpose?

A. CensusB. SurveyC. Record ReviewD. Review of civil registry

71. In the conduct of a census, the method of population assignment based on the actual physical location of the people is termed;

A. De jureB. De locusC. De factoD. De novo

72. The Field Health Services and information System (FHSIS) is the recording and reporting system in public health) care in the Philippines. The monthly field health service activity report is a form used in which of the components of the FHSIS?

A. Tally report B. Output reportC. Target/client listD. Individual health record

73. To monitor clients registered in long-term regimens, such as the Multi-Drug Therapy, which component will be most useful?

A. Tally report B. Output report C. Target/client list D. Individual health record

74. Civil registries are important sources of data. Which law requires registration of births within 30 days from the occurrence of the birth?

A. PD 651 B. Act 3573 C. RA 3753 D. RA 3375

75. Which of the following professionals can sign the birth certificate?

A. Public health nurse B. Rural health midwife C. Municipal health officerD. Any of these health professionals

76. Which criterion in priority setting of health problems is used only in community health care?

A. Modifiability of the problemB. Nature of the problem presentedC. Magnitude of the health problem D. Preventive potential of the health problem

77. The Sentrong Sigla Movement has been launched to improve health service delivery. Which of the following is/are true of this movement?

A. This is a project spearheaded by local government unitsB. It is a basis for increasing funding from local government unitsC. It encourages health centers to focus on disease prevention and controlD. Its main strategy is certification of health centers able to comply with standards

78. Which of the following women should be considered as special targets for family planning?A. Those who have two children or moreB. Those with medical conditions such as anemiaC. Those younger than 20 years and older than 35 yearsD. Those who just had a delivery within the past 15 months

79. Freedom of choice in one of the policies of the Family Planning Program of the Philippines. Which of the following illustrates this principle?

A. Information dissemination about the need for family planning B. Support of research and development in family planning methodsC. Adequate information for couples regarding the different methodsD. Encouragement of couples to take family planning as a joint responsibility

80. A woman, 6 months pregnant, came to the center for consultation. Which of the following substances is contraindicated?

Page 6: Document1

A. Tetanus toxoidB. Retinol 200,000 IUC. Ferrous sulfate 200mg D. Potassium iodate 200 mg, capsule

81. During prenatal consultation, a client asked you if she can have her delivery at home. After history taking and physical examination, you advised her against a home delivery. Which of the following findings disqualifies her for a home delivery?

A. Her OB score is G5P3B. She has some palmar pallor C. Her blood pressure is 130/80 D. Her baby is in cephalic presentation

82. Inadequate intake by the pregnant woman of which vitamin may cause neural tube defects?

A. NiacinB. RiboflavinC. Folic Acid D. Thiamine

83. You are in a client's home to attend to a delivery. Which of the following will you do first?

A. Set up a sterile area B. Put on a clean gown and apron C. Cleanse the client's vulva with soap and waterD. Note the interval, duration and intensity of labor and contractions

84. In preparing a primigravida for breastfeeding, which of the following will you do?

A. Tell her that lactation begins within a day after delivery B. Teach her nipple stretching exercises if her nipples are everted C. Instruct her to wash her nipples before and after each breastfeeding D. Explain to her that putting the baby to breast will lessen blood loss after delivery

85. A primigravida is instructed to offer her breast to the baby for the first time within 30 minutes after delivery. What is the purpose of offering the breast this early?

A. To initiate the occurrence of milk

letdown B. To stimulate milk production by the mammary acini C. To make sure that the baby is able to get the colustrum D. To allow the woman to practice breastfeeding in the presence of the health worker

86. In a mother's class, you discuss proper breastfeeding technique. Which of these is a sign that the baby has "lactated on" the breast property?

A. The baby takes shallow, rapid sucks B. The mother does not feel nipple pain C. The baby's mouth is only partly open D. Only the mother's nipple is inside the baby's mouth

87. You explain to a breastfeeding mother that breastmilk is sufficient for all of the baby's nutrient needs only up to:

A. 3 monthsB. 6 monthsC. 1 yearD. 2 years

88. What is given to a woman within a month after the delivery of a baby?

A. Malunggay capsule B. Ferrous sutfate l00mg O.D. C. Retinol 200.000 IU 1 capsule D. Potassium Iodate 200 mg, 1 capsule

89. Which biological used in EPI is stored in the freezer?

A. DPTB. Tetanus toxoidC. Measles vaccineD. Hepatitis B vaccine

90. Unused BCG should be discarded how many hours after reconstitution?

A. 2B. 4C. 6D. At the end of the day

91. In immunity school entrants with BCG, you not obliged to secure parental consent. This is because of which legal document?

A. PD 996

B. RA 7864C. Presidential Proclamation No. 6D. Presidential Proclamation No. 46

92. Which immunization produces a permanent scar?

A. DPTB. BCGC. Measles vaccination D. Hepatitis B vaccination

93. A 4 week old baby was brought to the health center for his first immunization. Which can be given to him?

A. DPT1B. OPV1C. Infant BCGD. Hepatitis B Vaccin

94. You will not give DPT 2 if the mother says that the infant had?

A. Seizures a day after DPT1B. Fever for 3 days after DPT1C. Abscess formation after DPT1D. Local tenderness for 3 days after DPT1

95. A 2-month old infant was brought to the health center for immunization. During assessment, the infant's temperature registered at 38.1 C. Which is the best course of action that you will take?

A. Go on with the infants immunizationB. Give paracetamol and wait for his fever to subsideC. Refer the infant to the physician for further assessmentD. Advise the infant's mother to bring him back for immunization when he is well

96. A pregnant woman had just received her 4th dose of tetanus toxoid. Subsequently, her baby will have protection against tetanus for how long?

A. 1 yearB. 3 yearsC. 10 yearsD. Lifetime

97. A 4-month old infant was brought to the health center of cough. Her respiratory rate is 42/minute. Using the IMCI guidelines of assessment, her breathing is considered;

Page 7: Document1

A. FastB. SlowC. NormalD. Insignificant

98. Which of the following signs will indicate that a young child is suffering from severe pneumonia?

A. DyspneaB. WheezingC. Fast breathingD. Chest indrawing

99. Using IMCI guidelines, you classify a child as having severe pneumonia. What is the best management for the child?

A. Prescribe antibioticB. Refer him urgently to the hospital C. Instruct the mother to increase fluid intake D. Instruct the mother to continue breastfeeding

100. A 5-month old infant was brought by his mother to the health center because of diarrhea occurring 4 to 5 times a day. His skin goes back slowly after a skin pinch and his eyes are sunken. Using the IMCI guidelines, you will classify this infant in which category?

A. No signs of dehydrationB. Some dehydrationC. Severe dehydration D. The data is insufficient

101. Based on the assessment, you classified a 3-month old infant with the chief complaint of diarrhea in the category of SOME DEHYDRATION. Based on the IMCI management guidelines, which of the following will you do?

A. Bring the infant to the nearest facility where IV fluids can be given B. Supervise the mother in giving 200 to 400ml of Oresol in 4 hoursC. Give the infant's mother instructions on home management D. Keep the infant in your health center for close observation

102. A mother is using Oresol' in the management of diarrhea of her 3-year old child. She asked you what to do if her child vomits. You will tell her to:

A. Bring the child to the nearest hospital for further assessment B. Bring the child to the health center for IV therapyC. Bring the child to the health center for assessment by the physicianD. Let the child rest for 10 minutes then continue giving Oresol more slowly

103. A 1 1/2 year old child was classified as having 3rd degree of protein energy malnutrition, kwashjorkor. Which of the following signs will be most apparent in this child?

A. Voracious appetiteB. WastingC. ApathyD. Edema

104. Assessment of a 2-year old child revealed "baggy pants". Using the IMCI guidelines, how will you manage this child?A. Refer the child urgently to a hospital for confinementB. Coordinate with the social worker to enroll the child in a feeding programC. Make a teaching plan for the mother, focusing on the menu planning for her childD. Assess and treat the child for health problems like infections and intestinal parasitism

105. During the physical examination of a young child, what is the earliest sign of xerophthalmia that may observe?

A. KeratomalaciaB. Corneal opacityC. Night blindnessD. Conjunctival xerosis

106. To prevent xerophthalmia, young children are given Retinol capsule every 6 months. What is the dose given to preschoolers?

A. 10, 000 IUB. 20, 000 IUC. 100, 000 IUD. 200, 000 IU

107. The major sign of iron deficiency anemia is pallor. What part is best examined for pallor?

A. PalmsB. NailbedsC. Around the lipsD. Lower conjunctival sac

108. Food fortification is one of the strategies to prevent micronutrient deficiency conditions. RA 8976 mandates fortification of certain food items. Which of the following is among these food items?

A. SugarB. BreadC. Margarine D. Filled milk

109. What is the best course of action when there is a measles epidemic in a nearby municipality?

A. Give measles vaccine to babies aged 6 to 3 months B. Give babies aged 6 to 11 months one dose of 100,000 IU of Retinol C. Instruct mother to keep their babies at home to prevent disease transmission D. Instruct mothers to feed their babies adequately to enhance their babies resistance

110. A mother brought her daughter, 4 years old, to the RHU because of cough and colds. Following the IMCI assessment guide, which of the following is a danger sign that indicates the need for urgent referral to a hospital?

A. Inability to drink B. High grade fever C. Signs of severe dehydration D. Cough for more than 30 days

111. Management of a child with measles includes the administration of which of the following?

A. Gentian violet on mouth lesionsB. Antibiotic to prevent pneumoniaC. Tetracycline eye ointment for corneal opacityD. Retinol capsule regardless of when the last dose was given

112. A mother brought her 10 month old infant for consultation because of fever which started 4 days prior to consultation. To determine malaria risk, what will you do?

Page 8: Document1

A. Do a tourniquet test B. Ask where the family resides C. Get a specimen for blood smear D. Ask if the fever is present everyday

113. The following are strategies implemented by the DOH to prevent mosquito-borne diseases. Which of these is most effective in the control of Dengue fever?

A. Stream seeding with larva-eating fishB. Destroying breeding places of mosquitoesC. Chemoprophylaxis of non-immune persons going to endemic areas D. Teaching people in endemic areas to use chemically treated mosquito nets

114. Secondary prevention for malaria includes?

A. Planting of neem or eucalyptus treesB. Residual spraying of insecticides at nightC. Determining whether a place is endemic or notD. Growing larva-eating fish in mosquito breeding places

115. Scotch tape swab is done to check for which intestinal parasite?

A. Ascaris B. Pinworm C. Hookworm D. Schistosoma

116. Which of the following signs indicates the need for sputum examination for AFB?

A. HematemesisB. Fever for 1 weekC. Cough for 3 weeksD. Chest pain for 1 week

117. Which clients are considered targets for DOTS category?

A. Sputum negative cavitary cases B. Clients returning after defaultC. Relapses and failures of previous PTB treatment regimens D. Clients diagnosed for the first time through a positive sputum exam

118. To improve compliance to treatment, what innovation is being

implemented in DOTS?

A. Having the health worker follow up the client at homeB. Having the health worker or a responsible family member monitor drug intake C. Having the patient come to the health center every month to get his medications D. Having a target list to check on whether the patient has collected his monthly supply of drugs

119. Diagnosis of leprosy is highly dependent on recognition of symptoms. Which of the following is an early sign of leprosy?

A. Macular lesions B. Inability to close eyelidsC. Thickened painful nervesD. Sinking of the nose bridge

120. Which of the following clients should be classified as a case of mutibacillary leprosy?

A. 3 skin lesions, negative slit skin smearB. 3 skin lesions, positive slit skin smear C. 5 skin lesions, negative slit skin smearD. 5 skin lesions, positive slit skin smear

121. In the Philippines, which condition is the most frequent cause of death associated by schistosomiasis?

A. Liver cancer B. Liver cirrhosis C. Bladder cancer D. Intestinal perforation

122. What is the most effective way of controlling schistosomiasis in an endemic area?

A. Use of molluscicidesB. Building of foot bridgesC. Proper use of sanitary toiletsD. Use of protective footwear, such as rubber boots

123. When residents obtain water from an artesian well in the neighborhood, the level of this approved type of water facility is:

A. IB. IIC. III

D. IV

124. For prevention of Hepatitis A, you decided to conduct health education activities. Which of the following is Irrelevant?

A. Use of sterile syringes and needles B. Safe food preparation and food handling by vendors C. Proper disposal of human excreta and personal hygiene D. Immediate reporting of water pipe leaks and illegal water connections

125. Which biological used in EPI should not be stored in the freezer?

A. DPTB. OPVC. Measles vaccineD. MMR

126. You will conduct outreach immunization in a barangay with a population of about 1500. Estimate the number of infants in the barangay.

A. 45B. 50C. 55D. 60

127. In IMCI, severe conditions generally require urgent referral to a hospital. Which of the following severe conditions Does not always require urgent referral to hospital?

A. MastoiditisB. Severe dehydration C. Severe pneumonia D. Severe febrile disease.

128. A client was diagnosed as having Dengue Fever. You will say that there is slow capillary refill when the color of the nailbed that you pressed does not return within how many seconds?

A. 3B. 5C. 8D. 10

129. A 3-year old child was brought by his mother to the health center because of fever of 4-day duration. The child had a positive tourniquet test result. In the

Page 9: Document1

absence of other signs, which of the most appropriate measure that the PHN may carry out to prevent Dengue shock syndrome?

A. Insert an NGT and give fluids per NGTB. Instruct the mother to give the child OresolC. Start the patient on IV StatD. Refer the client to the physician for appropriate management

130. The pathognomonic sign of measles is Koplik’s spot. You may see Koplik’s spot by inspecting the:

A. Nasal MucosaB. Buccal mucosa C. Skin on the abdomen D. Skin on the antecubital surface

131. Among the following diseases, which is airborne?

A. Viral conjunctivitisB. Acute poliomyelitisC. DiptheriaD. Measles

132. Among children aged 2 months to 3 years, the most prevalent form of meningitis is caused by which microorganism?

A. Hemophilus InfluenzaeB. MorbillivirusC. Streptococcus PneumoniaeD. Neisseria meningitides

133. Human beings are the major reservoir of malaria. Which of the following strategies in malaria control is based on this fact?

A. Stream seeding B. Stream clearingC. Destruction of breeding placesD. Zooprophylaxis

134. The use of larvivorous fish in malaria control is the basis for which strategy of malaria control?

A. Stream seedingB. Stream clearingC. Destruction of breeding places D. Zooprophylaxis .

135. Mosquito-borne diseases are

prevented mostly with the use of mosquito control measures. Which of the following is NOT appropriate for malaria control?

A. Use of chemically treated mosquito netsB. Seeding of breeding places with larva-eating fish C. Destruction of breeding places of the mosquito vectorD. Use of mosquito-repelling soaps, such as those with basil or citronella

136. A 4-year old client was brought to the health center with chief complaint of severe diarrhea and the passage of “rice water”. The client is most probably suffering from which condition?

A. GiardiasisB. Cholera C. Amebiasis D. Dysentery

137. In the Philippines, which specie of schistosoma is endemic in certain regions?

A. S. mansoniB. S. japonicumC. S. malayensisD. S. haematobium

138. A 32 year old client came for consultation at the health center with the chief complaint of fever for a week. Accompanying symptoms were muscle pains and body malaise. A week after the start of fever, the client noted yellowish discoloration of his sclera. History showed that he waded in flood waters about 2 weeks before the onset of symptoms. Based on this history/ which disease condition will you suspect?

A. Hepatitis AB. Hepatitis BC. TetanusD. Leptospirosis

139. MWSS provides water to Manila and other cities in Metro Manila. This is an example of which level of water facility?

A. IB. IIC. III D. IV

140. You are the PHN in the city health center. A client underwent screening for AIDS using ELISA. His result was positive. What is the best course of action that you may take?

A. Get a thorough history of the client, focusing on the practice of high risk behavior B. Ask the client to be accompanied by a significant person before revealing the result.C. Refer the client to the physician since he is the best person to reveal the result to the client D. Refer the client for a supplementary test, such as Western blot, since the ELISA result maybe false

141. Which is the BEST control measure for AIDS?

A. Being faithful to a single sexual partner B. Using a condom during each sexual contact C. Avoiding sexual contact with commercial sex workers D. Making sure that one's sexual partner does not have signs of AIDS

142. The most frequent causes of death among clients with AIDS are opportunistic diseases. Which of the following opportunistic infections is characterized by tonsilllopharyngitis?

A. Respiratory candidiasisB. Infectious mononucleosis C. Cytomegalovirus disease D. Pneumocystis carinii pneumonia

143. To determine the possible sources of sexually transmitted infections, which is the BEST method that may be undertaken by the public health nurse?

A. Contact tracing B. Community survey C. Mass screening testsD. Interview suspects

144. Antiretroviral agents, such as AZT are used in the management of AIDS. Which of the following is not an action expected of these drugs?

A. They prolong the life of the client with AIDSB. They reduce the risk of opportunistic

Page 10: Document1

infectionsC. They shorten the period of communicability of the disease D. They are able to bring about a cure of the disease condition

145. A barangay had an outbreak of German measles. To prevent congenital rubella, what is the BEST advice that you can give to women in the first trimester of pregnancy in the barangay?

a. Advice them on the sign of German Measlesb. Avoid crowded places, such as markets and moviehousesc. Consult at the health center where rubella vaccine may be givend. Consult a physician who may give them rubella immunoglobulin

Page 11: Document1

FOUNDATION OF PROFESSIONAL NURSING PRACTICE

Situation 1 - Mr. Ibarra is assigned to the triage area and while on duty, he assesses the condition of Mrs. Simon who came in with asthma. She has difficulty breathing and her respiratory rate is 40 per minute. Mr. Ibarra is asked to inject the client epinephrine 0.3mg subcutaneously

1. The indication for epinephrine injection for Mrs Simon is to:

a. Reduce anaphylaxisb. Relieve hypersensitivity to allergenc. Relieve respirator distress due to bronchial spasmd. Restore client’s cardiac rhythm

2. When preparing the epinephrine injection from an ampule, the nurse initially:

a. Taps the ampule at the top to allow fluid to flow to the base of the ampuleb. Checks expiration date of the medication ampulec. Removes needle cap of syringe and pulls plunger to expel aird. Breaks the neck of the ampule with a gauze wrapped around it

3. Mrs. Simon is obese. When administering a subcutaneous injection to an obese patient, it is best for the nurse to:

a Inject needle at a 15 degree angle' over the stretched skin of the clientb. Pinch skin at the Injection site and use airlock techniquec. Pull skin of patient down to administer the drug in a Z trackd. Spread skin or pinch at the injection site and inject needle at a 45-90 degree angle

4. When preparing for a subcutaneous injection, the proper size of syringe and needle would be:

a. Syringe 3-5ml and needle gauge 21 to 23b. Tuberculin syringe 1 mi with needle gauge 26 or 27c. Syringe 2ml and needle gauge 22d. Syringe 1-3ml and needle gauge 25 to 27

5. The rationale for giving medications through the subcutaneous route is;

a. There are many alternative sites for subcutaneous injectionb. Absorption time of the medicine is slowerc. There are less pain receptors in this aread. The medication can be injected while the client is in any position

Situation 2 - The use of massage and meditation to help decrease stress and pain have been strongly recommended based on documented testimonials.

6. Martha wants to do a study on, this topic. "Effects of massage and meditation on stress and pain." The type of research that best suits this topic is:

a. applied researchb. qualitative researchc. basic researchd. quantitative research

7. The type of research design that does not manipulate independent variable is:

a. experimental designb. quasi-experimental designc. non-experimental designd. quantitative design

8. This research topic has the potential to contribute to nursing because it seeks to:

a. include new modalities of careb. resolve a clinical problemc. clarify an ambiguous modality of cared. enhance client care

9. Martha does review of related literature for the purpose of:

a. determine statistical treatment of data researchb. gathering data about what is already known or unknownc. to identify if problem can be replicatedd. answering the research question

10. Client’s rights should be protected when doing research using human subjects. Martha identifies these rights as follows EXCEPT:

a. right of self-determinationb. right to compensation

c. right of privacyd. right not to be harmed

Situation 3 - Richard has a nursing diagnosis of ineffective airway clearance related to excessive secretions and is at risk for infection because of retained secretions. Part of Nurse Mario's nursing care plan is to loosen and remove excessive secretions in the airway,

11. Mario listens to Richard's bilateral sounds and finds that congestion is in the upper lobes of the lungs. The appropriate position to drain the anterior and posterior apical segments of the lungs when Mario does percussion would be:

a. Client lying on his back then flat on his abdomen on Trendelenburg positionb. Client seated upright in bed or on a chair then leaning forward in sitting position then flat on his back and on his abdomenc. Client lying flat on his back and then flat on his abdomend. Client lying on his right then left side on Trendelenburg position

12. When documenting outcome of Richard's treatment Mario should include the following in his recording EXCEPT:

a. Color, amount and consistent of sputumb. Character of breath sounds and respirator/rate before and after procedurec. Amount of fluid intake of client before and after the procedured. Significant changes in vital signs

13. When assessing Richard for chest percussion or chest vibration and postural drainage Mario would focus on the following EXCEPT:

a. Amount of food and fluid taken during the last meal before treatmentb. Respiratory rate, breath sounds and location of congestionc. Teaching the client's relatives to perform 'the procedured. Doctor's order regarding position restriction and client's tolerance for lying flat

14. Mario prepares Richard for postural drainage and percussion. Which of the flowing is a special consideration when doing the procedure?

Page 12: Document1

a. Respiratory rate of 16 to 20 per minuteb. Client can tolerate sitting and lying positionc. Client has no signs of infectiond. Time of fast food and fluid intake of the client

15. The purpose of chest percussion and vibration is to loosen secretions in the lungs. The difference between the procedure is;

a. Percussion uses only one hand white vibration uses both handsb. Percussion delivers cushioned blows to the chest with cupped palms while gently shakes secretion loose on the exhalation cyclec. In both percussion and vibration the hands are on top of each other and hand action is in tune with client's breath rhythmd. Percussion slaps the chest to loosen secretions while vibration shakes the secretions along with the inhalation of air

Situation 4 - A 61 year old man, Mr. Regalado, is admitted to the private ward for observation; after complaints of severe chest pain. You are assigned to take care of the client.

16. When doing an initial assessment, the best way for you to identify the client’s priority problem is to:

a. Interview the client for chief complaints and other symptomsb. Talk to the relatives to gather data about history of illnessc. Do auscultation to check for chest congestiond. Do a physical examination white asking the client relevant questions

17. Upon establishing Mr. Regalado's nursing needs, the next nursing approach would be to:

a. introduce the client to the ward staff to put the client and family at easeb. Give client and relatives a brief tour of the physical set up the unitc. Take his vital signs for a baseline assessmentd. Establish priority needs and implement appropriate interventions

18. Mr. Regalado says he has "trouble

going to sleep". In order to plan your nursing intervention you will.

a. Observe his sleeping patterns in the next few daysb. Ask him what he means by this statementc. Check his physical environment to decrease noise leveld. Take his blood pressure before sleeping and upon waking up

19. Mr. Regalado's lower extremities are swollen and shiny. He has pitting pedal edema. When taking care of Mr. Regalado, which of the following intervention would be the most appropriate immediate nursing approach.

a. Moisturize lower extremities to prevent skin irritationb. Measure fluid intake and output to decrease edemac. Elevate lower extremities for postural drainaged. Provide the client a list of food low in sodium

20. Mr. Regalado will be discharged from your unit within the hour. Nursing actions when preparing a client for discharge include all EXCEPT:

a. Making a final physical assessment before client leaves the hospitalb. Giving instructions about his medication regimenc. Walking the client to the hospital exit to ensure his safetyd. Proper recording of pertinent data

Situation 5 - Nancy, mother of 2 young kids. 36 years old, had a mammogram and was told that she has breast cysts and that she may need surgery. This causes her anxiety as shown by increase in her pulse and respiratory rate, sweating and feelings of tension.

21. Considering her level of anxiety, the nurse can best assist Nancy by:

a. Giving her activities to divert her attentionb. Giving detailed explanations about the treatments she will undergoc. Preparing her and her family in case surgery is not successfuld. Giving her clear but brief information at

the level of her understanding

22. Nancy blames God for her situation. She is easily provoked to tears and wants to be left alone, refusing to eat or talk to her family. A religious person before, she now refuses to pray or go to church stating that God has abandoned her. The nurse understands that Nancy is grieving for her self and is in the stage of:

a. bargainingb. denialc. angerd. acceptance

23. The nurse visits Nancy and prods her to eat her food. Nancy replies "what's the use? My time is running out. The nurse's best response would be:

a. "The doctor ordered full diet for you so that you will be strong for surgery."b. "I understand how you fee! but you have 1o try for your children's sake."c. "Have you told your, doctor how you feel? Are you changing your mind) about surgery?"d. "You sound like you are giving up."

24. The nurse feels sad about Nancy's illness and tells her head nurse during the end of shift endorsement that "it's unfair for Nancy to have cancer when she is still so young and with two kinds. The best response of the head nurse would be:

a. Advise the nurse to "be strong and learn to control her feelings"b. Assign the nurse to another client to avoid sympathy for the clientc. Reassure the nurse that the client has hope if she goes through all statements prescribed for herc. Ask the other nurses what they feel about the patient to find out if they share the same feelings

25. Realizing that she feels angry about Nancy's condition, the nurse Seams that being self-aware is a conscious process that she should do in any situation like this because:

a. This is a necessary part of the nurse -client relationship processb. The nurse is a role model for the client and should be strongC. How the nurse thinks and feels affect

Page 13: Document1

her actions towards her client and her workd. The nurse has to be therapeutic at all times and should not be affected

Situation 6 – Mrs. Seva, 32 years old, asks you about possible problems regarding her elimination now that she is in the menopausal stage.

26. Instruction on health promotion regarding urinary elimination is important. Which would you include?

a. Hold urine, as long as she can before emptying the bladder to strengthen her sphincters musclesb. If burning sensation is experienced while voiding, drink pineapple-juicec. After urination, wipe from anal area up towards the pubisd. Jell client to empty the bladder at each voiding

27. Mrs. Seva also tells the nurse that she is often constipated. Because she is aging, what physical changes predispose her to constipation?

a. inhibition of the parasympathetic reflexb. weakness of sphincter muscles of the anusc. loss of tone of the smooth muscles of the colord. decreased ability to absorb fluids in the lower intestines

28. The nurse understands that one of these factors contributes to constipation:

a. excessive exerciseb. high fiber dietc. no regular tine for defecation dailyd. prolonged use of laxatives

29. Mrs. Seva talks about rear of being incontinent due to a prior experience of dribbling urine when laughing or sneezing and when she has a full bladder. Your most appropriate .instruction would be to:

a. tell client to drink less fluids to avoid accidentsb. instruct client to start wearing thin adult diapersc. ask the client to bring change of underwear "just in case"d. teach client pelvic exercise to

strengthen perineal muscles

30. Mrs. Seva asked for instructions for skin care for her mother who has urinary incontinence and is almost always in bed. Your instruction would focus on prevention of skin irritation and breakdown by

a. Using thick diapers to absorb urine wellb. Drying the skin with baby powder to prevent or mask the smell of ammoniac. Thorough washing, rising and during of skin area that get wet with urined. Making sure that linen are smooth and dry at all times

Situation 7 - Using Maslow's need theory, Airway, Breathing and Circulation are the physiological needs vital to life. The nurse's knowledge and ability to identify and immediately intervene to meet these needs is important to save lives.

31. Which of these clients has a problem with the transport of oxygen from the lungs to the tissues:

a. Carol with a tumor in the brainb. Theresa with anemiac. Sonny Boy with a fracture in the femurd. Brigette with diarrhea

32. You noted from the lab exams in the chart of Mr. Santos that he has reduced oxygen in the blood.This condition is called:

a. Cyanosisb. Hypoxiac. Hypoxemiad. Anemia

33. You will nasopharyngeal suctioning Mr. Abad. Your guide for the length of insertion of the tubing for an adult would be:

a. tip of the nose to the base of the .neckb. the distance from the tip of the nose to the middle of the cheekc. the distance from the tip of the nose to the tip of the ear lobed. eight to ten inches

34. While doing nasopharyngeal suctioning on .Mr. Abad, the nurse can avoid trauma to the area by:

a. Apply suction for at least 20-30 seconds each time to ensure that all secretions are removedb. Using gloves to prevent introduction of pathogens to the respiratory systemc. Applying no suction while inserting the catheterd. Rotating catheter as it is inserted with gentle suction

35. Myrna has difficulty breathing when on her back and must sit upright in bed to breath, effectively and comfortably. The nurse documents this condition as:

a. Apneab. Orthopneac. Dyspnead. Tachypnea

Situation 8 - You are assigned to screen for hypertension: Your task is to take blood pressure readings and you are informed about avoiding the common mistakes in BP taking that lead to 'false or inaccurate blood pressure readings.

36. When taking blood pressure reading the cuff should be:

a. deflated fully then immediately start second reading for same clientb deflated quickly after inflating up to 180 mmHgc. large enough to wrap around upper arm of the adult client 1 cm above brachial arteryd. inflated to 30 mmHg above the estimated systolic BP based on palpation of radial or bronchial artery

37. Chronic Obstructive Pulmonary Disease (COPD) in one of the leading causes of death worldwide and is a preventable disease. The primary cause of COPD is:

a. tobacco hackb. bronchitisc. asthmad. cigarette smoking

38. In your health education class for clients with diabetes you teach, them the areas, for control . Diabetes which include all EXCEPT:

a. regular physical activityb. thorough knowledge of foot care

Page 14: Document1

c. prevention nutritiond. proper nutrition

39. You teach your clients the difference between, Type I (IDDM) and Type II (NDDM) Diabetes. Which of the following is true?

a. both types diabetes mellitus clients are all prone to developing ketosisb. Type II (NIDDM) is more common and is also preventable compared to Type I (IDDM) diabetes which is genetic in etiologyc. Type I (IDDM) is characterized by fasting hyperglycemiad. Type II (IDDM) is characterized by abnormal immune response

40. Lifestyle-related diseases in general share areas common risk factors. These are the following excepta. physical activityb. smokingc. geneticsd. nutrition

Situation 9 - Nurse Rivera witnesses a vehicular accident near the hospital where she works. She decides to get involved and help the victims of the accident.

41. Her priority nursing action would be to:

a. Assess damage to propertyb. Assist in the police investigation since she is a witnessc. Report the incident immediately to the local police authoritiesd. Assess the extent of injuries incurred by the victims, of the accident

42. Priority attention should be given to which of these clients?

a. Linda who shows severe anxiety due to trauma of the accidentb. Ryan who has chest injury, is pate and with difficulty of breathingc. Noel who has lacerations on the arms with mild-bleedingc. Andy whose left ankle swelled and has some abrasions

43. In the emergency room, Nurse Rivera is assigned to attend to the client with .lacerations on the arms, while assessing the extent of the wound the

nurse observes that the wound is now starting to bleed profusely. The most immediate nursing action would be to:

a. Apply antiseptic to prevent infectionb. Clean the wound vigorously of contaminantsc. Control and. reduce bleeding of the woundd. Bandage the wound and elevate the arm

44. The nurse applies pressure dressing on the bleeding site. This intervention is done to:

a. Reduce the need to change dressing frequentlyb. Allow the pus to surface fasterc. Protect the wound from micro organisms in the aird. Promote hemostasis

45. After the treatment, the client is sent home and asked to come back for follow-up care. Your responsibilities when the client is to be discharged include the following EXCEPT:

a. Encouraging the client to go to the, outpatient clinic for follow up careb. Accurate recording, of treatment done and instructions given to clientc. Instructing the client to see you after discharge for further assistanced. Providing instructions regarding wound care

Situation 10 - While working in the clinic, a new client, Geline, 35 years old, arrives for her doctor's appointment. As the clinic nurse, you are to assist the client fiil up forms, gather data and make an assessment.

46. The nurse purpose of your initial nursing interview is to:

a. Record pertinent information in the client chart for health team to readb Assist the client find solutions to her health concernsc. Understand her lifestyle, health needs and possible problems to develop a plan of cared. Make nursing diagnoses for identified health problems

47. While interviewing Geline, she starts

to moan and doubles up in pain, She tells you that this pain occurs about an hour after taking black coffee without breakfast for a few weeks now. You will record this as follows:

a. Claims to have abdominal pains after intake of coffee unrelieved by analgesicsb. After drinking coffee, the client experienced severe abdominal painc. Client complained of intermittent abdominal pain an hour after drinking coffeed. Client reported abdominal pain an hour after drinking black coffee for three weeks now

48. Geline tells you that she drinks black coffee frequently within the day to "have energy and be wide awake" and she eats nothing for breakfast and eats strictly vegetable salads for lunch and dinner to lose weight. She has lost weight during the past two weeks, in planning a healthy balanced diet with Geline, you will:

a. Start her off with a cleansing diet to free her body of toxins then change to a vegetarian, diet and drink plenty of fluidsb. Plan a high protein, diet; low carbohydrate diet for her considering her favorite foodc. Instruct her to attend classes in nutrition to find food rich in complex carbohydrates to maintain daily high energy leveld. Discuss with her the importance of eating a variety of food from the major food groups with plenty of fluids

49. Geline tells you that she drinks 4-5 cups of black coffee and diet cola drinks. She also smokes up to a pack of cigarettes daily. She confesses that she is in her 2nd month of pregnancy but she does not want to become fat that is why she limits her food intake. You warn or caution her about which of the following?

a. Caffeine products affect the central nervous system and may cause the mother to have a "nervous breakdown"b. Malnutrition and its possible effects on growth and development problems in the unborn fetusc. Caffeine causes a stimulant effect on both the mother and the babyd. Studies show conclusively that caffeine causes mental retardation

Page 15: Document1

50. Your health education plan for Geline stresses proper diet for a pregnant woman and the prevention of non-communicable diseases that are influenced by her lifestyle these include of the following EXCEPT:

a. Cardiovascular diseasesb. Cancerc. Diabetes Mellitusd. Osteoporosis

Situation 11 - Management of nurse practitioners is done by qualified nursing leaders who have had clinical experience and management experience.

51. An example of a management function of a nurse is:

a. Teaching patient do breathing and coughing exercisesb. Preparing for a surprise party for a clientc. Performing nursing procedures for clientsd. Directing and evaluating the staff nurses

52. Your head nurse in the unit believes that the staff nurses are not capable of decision making so she makes the decisions for everyone without consulting anybody. This type of leadership is:

a. Laissez faire leadershipb. Democratic leadershipc. Autocratic leadershipd. Managerial leadership

53. When the head nurse in your ward plots and approves your work schedules and directs your work, she is demonstrating:

a. Responsibilityb. Delegationc. Accountabilityd. Authority

54. The following tasks can be safely delegated' by a nurse to a non-nurse health worker EXCEPT:

a. Transfer a client from bed to chair b. Change IV infusionsc. Irrigation of a nasogastric tubed. Take vital signs

55. You made a mistake in giving the medicine to the wrong client You notify

the client’s doctor and write an incident report. You are demonstrating:

a. Responsibilityb. Accountabilityc. Authorityd. Autocracy

Situation 12 – Mr. Dizon, 84 years old, is brought to the .Emergency Room for complaint of hypertension flushed face, severe headache, and nausea. You are doing the initial assessment of vital signs.

56. You are to measure the client’s initial blood pressure reading by doing all of the following EXCEPT:

a. Take the blood pressure reading on both arms for comparisonb. Listen to and identify the phases of Korotkoff’s soundsc. Pump the cuff up to around 50 mmHg above the point where the pulse is obliteratedd. Observe procedures for infection control

57. A pulse oximeter is attached to Mr. Dizon’s finger to:

a. Determine if the client’s hemoglobin level is low and if he needs blood transfusionb. Check level of client’s tissue perfusionc. Measure the efficacy of the client’s anti hypertensive medicationsd. Detect oxygen saturation of arterial blood before symptoms of hypoxemia develops

58. After a few hours in the Emergency Room, Mr. Dizon is admitted to the ward with an order of hourly monitoring of blood pressure. The nurse finds that the cuff is too narrow and this will cause the blood pressure reading to be:

a. Inconsistentb. low systolic and high diastolic pressurec. higher than what the reading should bed. lower than what the reading should be

59. Through the client’s health history, you gather that Mr. Dizon smokes and drinks coffee. When taking the blood pressure of a client who recently smoked or drank coffee, how long should be the nurse wait before taking the client’s

blood pressure for accurate reading?

a. 15 minutesb. 30 minutesc. 1 hourd. 5 minutes

60. While the client has the pulse oximeter on his fingertip, you notice that the sunlight is shining on .the area where the oximeter is. Your action will be to:

a. Set and turn on the alarm of the oximeterb. Do nothing since there is no identified problemc. Cover the fingertip sensor with a towel or bedsheetd. Change the location of the sensor every four hours

Situation 13 - The nurse's understanding of ethico-legal responsibilities will guide his/her nursing practice.

61. The principles that .govern right and proper conducts of a person regarding life, biology and the health professions is referred to as:

a. Moralityb. Religionc. Valuesd. Bioethics

62. The purpose of having nurses’ code of ethics is:

a. Delineate the scope and areas of nursing practiceb. Identify nursing action recommended for specific healthcare situationsc. To help the public understand professional conduct, expected of nursesd. To define the roles and functions of the health care giver, nurses, clients

63. The most important nursing responsibility where ethical situations emerge in patient care is to:

a. Act only when advised that the action is ethically soundb. Not take sides remain neutral and fairc. Assume that ethical questions are the responsibility: of the health teamd. Be accountable for his or her own actions

Page 16: Document1

64. You inform the patient about his rights which include the following EXCEPT:

a. Right to expect reasonable continuity of careb. Right to consent to or decline to participate in research studies or experimentsc. Right to obtain information about another patientd. Right to expect that the records about his care will be treated as confidential

65. The principle states that a person has unconditional worth and has the capacity to determine his own destiny.

a. Bioethicsb. Justicec. Fidelityd. Autonomy

Situation 14 – Your director of nursing wants to improve the quality of health care offered in the hospital. As a staff nurse in that hospital you know that this entails quality assurance programs.

66. The following mechanisms can be utilized as part of the quality assessment program of your hospital EXCEPT:

a. Patient satisfaction surveys providedb. Peer review clinical records of care of clientc. RO of the Nursing Intervention Classificationd.

67. The nurse of the Standards of Nursing Practice is important in the hospital. Which of the following statements best describes what it is?

a. These are statements that describe the maximum or highest level of acceptable performance in nursing practice.b. It refers to the scope of nursing as defined in Republic Act 9173c. It is a license issued by the Professional Regulation Commission to protect the public from substandard nursing practice.d. The Standards of care includes the various steps of the nursing process and the standards of professional performance.

68. You are taking care of critically ill

client and the doctor in charge calls to order a DNR (do not resuscitate) for the client. Which of the following is the appropriate action when getting DNR order over the phone?

a. Have the registered nurse, family spokesperson, nurse supervisor and doctor signb. Have two nurses validate the phone order, both nurses sign the order and the doctor should sign his order within 24 hours.c. Have the registered nurse, family and doctor sign the orderd. Have 1 nurse take the order and sign it and have the doctor sign it within 24 hours

69. To ensure the client safety before starting blood transfusion the following are needed before the procedure can be done EXCEPT:

a. take baseline vital signsb. blood should be warmed to room temperature for 30 minutes before blood transfusion is administeredc. have two nurses verify client identification, blood type, unit number and expiration date of bloodd. get a consent signed for blood transfusion

70. Part of standards of care has to do with the use of restraints. Which of the following statements is NOT true?

a. Doctor’s order for restraints should be signed within 24 hoursb. Remove and reapply restraints every two hoursc. Check client’s pulse, blood pressure and circulation every four hoursd. Offer food and toileting every two hours

Situation 15 – During the NUTRITION EDUCATION class discussion a 58 year old man, Mr. Bruno shows increased interest.

71. Mr. Bruno asks what the "normal" allowable salt intake is. Your best response to Mr. Bruno is:

a. 1 tsp of salt/day with iodine and sprinkle of MSGb. 5 gms per day or 1 tsp of table salt/dayc. 1 tbsp of salt/day with some patis and toyod. 1 tsp of salt/day but not patis or toyo

72. Your instructions to reduce or limit salt intake include all the following EXCEPT:a. eat natural food with little or no salt addedb. limit use of table salt and use condiments insteadc. use herbs and spicesd. limit intake of preserved or processed food

73. Teaching strategies and approaches when giving nutrition education is influenced by age, sex and immediate concerns of the group. Your presentation for a group of young mothers would be best if you focus on:

a. diets limited in salt and fatb. harmful effect on drugs and alcohol intakec. commercial preparation of dishesd. cooking demonstration and meal planning

74. Cancer cure is dependent on

a. use of alternative methods of healingb. watching out for warning signs of cancerc. proficiency in doing breast self-examinationd. early detection and prompt treatment

75. The role of the health worker in health education is to:

a. report incidence of non-communicable disease to community health centerb. educate as many people about warning signs of non-communicable diseasesc. focus on smoking cessation projectsd. monitor clients with hypertension

Situation 16 – You are assigned to take care of 10 patients during the morning shift. The endorsement includes the IV infusion and medications for these clients.

76. Mr. Felipe, 36 years old is to be given 2700ml of D5RL to infuse for 18 hours starting at 8am. At what rate should the IV fluid be flowing hourly?

a. 100 ml/hourb. 210 ml/hourc. 150 ml/hourd. 90 ml/hour

Page 17: Document1

77. Mr. Atienza is to receive 150mg/hour of D5W IV infusion for 12 hours for a total of 1800ml. He is also losing gastric fluid which must be replaced every two hours. Between 8am to 10am. Mr. Atienza has lost 250ml of gastric fluid. How much fluid should he receive at 11am?

a. 350 ml/hourb. 275 ml/hourc. 400 ml/hourd. 200 ml/hour

78. You are to apply a transdermal patch of nitroglycerin to your client. The following important guidelines to observe EXCEPT:

a. Apply to hairlines clean are of the skin not subject to much wrinklingb. Patches may be applied to distal part of the extremities like forearmc. Change application and site regularly to prevent irritation of the skind. Wear gloves to avoid any medication of your hand

79. You will be applying eye drops to Miss Romualdez. After checking all the necessary information and cleaning the affected eyelid and eyelashes you administer the ophthalmic drops by instilling the eye drops.

a. directly onto the corneab. pressing on the lacrimal ductc. into the outer third of the lower conjunctival sacd. from the inner canthus going towards the side of the eye

80. When applying eye ointment, the following guidelines apply EXCEPT:

a. squeeze about 2 cm of ointment and gently close but not squeeze eyeb. apply ointment from the inner canthus going outward of the affected eyec. discard the first bead of the eye ointment before application because the tube likely to expel more than desired amount of ointmentd. hold the tube above the conjunctival sac do not let tip touch the conjuctiva

Situation 17 – The staff nurse supervisor request all the staff nurses to “brainstorm” and learn ways to instruct diabetic clients on self-administration of insulin. She wants

to ensure that there are nurses available daily to do health education classess.

81. The plan of the nurse supervisor is an example of

a. in service education processb. efficient management of human resourcesc. increasing human resourcesd. primary prevention

82. When Mrs. Guevarra, a nurse, delegates aspects of the clients care to the nurse-aide who is an unlicensed staff, Mrs. Guevarra.

a. makes the assignment to teach the staff memberb. is assigning the responsibility to the aide but not the accountability for those tasksc. does not have to supervise or evaluate the aided. most know how to perform task delegated

83. Connie, the-new nurse, appears tired and sluggish and lacks the enthusiasms she give six weeks ago when she started the job. The nurse supervisor should:

a. empathize with the nurse and listen to her b. tell her to take the day offc. discuss how she is adjusting to her new jobd. ask about her family life

84. Process of formal negotiations of working conditions between a group of registered nurses and employer is:

a. grievanceb. arbitrationc. collective bargainingd. strike

85. You are attending a certification program on cardiopulmonary resuscitation (CPR) offered and required by the hospital employing you. This is;

a. professional course towards creditsb. in-service educationc. advance training d. continuing education

Situation 18 - There are various developments in health education that the

nurse should know about.

86. The provision of health information in the rural areas nationwide through television and radio programs and video conferencing is referred to as:

a. Community health programb. Telehealth programc. Wellness program d. Red cross program

87. A nearby community provides blood pressure screening, height and weight measurement smoking cessation classes and aerobics class services. This type of program is referred to as:

a. outreach programb. hospital extension programc. barangay health center d. wellness center

88. Part of teaching client in health promotion is responsibility for one’s health. When Danica states she need to improve her nutritional status this means:

a. Goals and interventions to be followed by client are based on nurse's prioritiesb. Goals and intervention developed by nurse and client should be approved by the doctorc. Nurse will decide goals and, interventions needed to meet client goalsd. Client will decide the goals and interventions required to meet her goals

89. Nurse Beatrice is providing tertiary prevention to Mrs. De Villa. An example of tertiary provestion is:

a. Marriage counselingb. Self-examination for breast cancerc. Identifying complication of diabetesd. Poison, control

90. Mrs. Ostrea has a schedule for Pap Smear. She has a strong family history of cervical cancer. This is an example of:

a. tertiary preventionb. secondary preventionc. health screeningd. primary prevention

Situation: 19 - Ronnie has a vehicular accident where he sustained injury to his left ankle. In the Emergency Room, you

Page 18: Document1

notice how anxious he looks.

91. You establish rapport with him and to reduce his anxiety you initially

a. Take him to the radiology, section for X-ray of affected extremityb. Identify yourself and state your purpose in being with the clientc. Talk to the physician for an order of

Valiumd. Do inspection and palpation to check extent of his injuries

92. While doing your assessment, Ronnie asks you "Do I have a fracture? I don't want to have a cast.” The most appropriate nursing response would be:

a. "You have to have an X-ray first to know

if you have a fracture."b. "Why do you; sound so scared? It is just a cast and it's not painful"c. "You seem to be concerned about being in a cast." d. "Based on my assessment, there doesn’t seem to be a fracture."

ANSWER KEY1. C2. B3. D4. D5. B6. B7. C8. D9. B10. B11. B12. C13. C14. D15. A16. A17. C18. B19. A20. C21. D22. C23. D24. D25. C26. D27. C28. D29. D30. C

31. B32. C33. C34. C35. B36. D37. D38. B39. B40. C41. D42. B43. D44. D45. C46. C47. D48. D49. B50. D51. D52. C53. D54. B55. B56. C57. D58. C59. B60. C61. D

62. C63. D64. C65. D66. D67. A68. D69. D70. C71. B72. B73. D74. D75. B76. C77. -78. B79. B80. C81. C82. B83. C84. C85. B86. B87. A88. D89. C90. B91. B92. C

Page 19: Document1

FUNDAMENTAL OF NURSING

1. The nurse knows which of the following indicates protein deficiency?

a. Hypoalbuminemia and pot bellyb. Koionychia (spoon-shaped nails)c. Beefy red tongued.d. Bleeding gums

2. Before administering a tube feeding the nurse knows to perform which of the following assessments?

a. The GI tract including bowel sound, last BM and distentionb. The clients neurologic status, especially gag reflexc. The amount of air in the stomachd. The formula used directly from the refrigerator

3. An adult is being taught about a healthy diet. The nurse can offer:

a. Milkb. Jelloc. Freshly squeezed orange juiced. Ice cream

5. An adult has received an injection of immunoglobulin. The nurse knows that the client will develop which of the following types of immunity:

a. Active natural immunityb. Active artificial immunityc. Passive natural immunityd. Passive artificial immunity

6. BON Resolution No 432 S 2003 is:

a. Guidelines for the implementation of Board Resolution No. 14 S 1999b. Implementing Rules and Regulations of RA 9173c. Special training on intravenous injection for nursesd. Adoption of a Guide to Evaluate Compliance with Standard for Safe Nursing Practice.

7. Its a type of non-probability sampling wherein the researcher identifies population strata and determines how many participants are needed for each stratum

a. Quota Samplingb. Purposive Samplingc. Convenience samplingd. Simple stratified sampling

8. All but one are concepts of informed consent.

a. It must be voluntary-without coercion or forceb. Patient must be competentc. Knowledgeable information should be providedd. It must be signed right after administration of pre-ops meds.

9. The following concepts of liability negligence are true, except?

a. Deviation from standard of care is established.b. Duty is owed to the nursec. Financial, physical and emotional harm is establishd. Direct cause for failure to meet standard of care clearly established.

10. The nurse is caring for a client whose arterial blood gases indicate metabolic acidosis. The nurse knows that of the following, the least likely to cause metabolic acidosis is:

a. cardiac arrestb. Diabetic ketoacidosisc. decreased serum potassium leveld. renal failure

11. The nurse is caring for a client who is receiving IV fluids, Which observation the nurse makes best indicates that the IV has infiltrated?

a. Pain at the siteb. A change in flow ratec. Coldness around the insertion sited. Redness around the insertion site

12. A 27 y.o adult is admitted for treatment of Crohn's disease. Which information is most significant when the nurse assesses nurtritional health?

a. Anthropometric measurementsb. bleeding gumsc. dry skind. facial rubor

13. ASA (aspirin) is being administered to

a client. The nurse understands that the most common mechanism of action for nonnarcotic analgesic is their ability to:

a. Inhibit prostaglandin systhesisb. After pain perception in the cerebellumc. Directly affect the central nervous systemd. Target the pain-producing effect of kinins

14. The nurse caring for an adult client who is receiving TPN will need to be monitored for which of the following metabolic complications?

a. Hypoglycemia and Hypercalcemiab. Hyperglycemia and Hypokalemiac. Hyperglycemia and Kyperkalemiad. Hyperkalemia and Hypercalcemia

15. Total parenteral nutrition is ordered for an adult. Which nutrient is not likely to be in the solution?

a. Dextrose 10%b. Trace mineralsc. Amino acidsd. Non of the above

16. A man has sprained his ankle. The physician would order cold applied to the injured area to.

a. Reduce the body's temperatureb. Increase circulation to the areac. Aid in absorbing the edemad. Relieve pain and control bleeding.

17. An adult is to have a tepid sponge bath to lower his fever. What temperature should the nurse make the water?

a. 65 Fb. 90 Fc. 110 Fd. 105 F

18. An adult has chronic lower back pain and receives hot pack three times a week. The nurse knows that the treatment is given for which of the following reasons?

a. To help remove debris from the woundb. To keep the client warm and raise his temperaturec. To improve the client's general circulation

Page 20: Document1

d. To relieve muscle spasm and promote muscle relaxation

19. An insane man got hold of a knife and stabbed a pregnant woman. His liability will be determined by virtue of :

a. Justifying circumstancesb. Exempting circumstancesc. Mitigating circumstancesd. Aggravating circumstances

20. A crime in which all the elements necessary for its execution and accomplishment are present.

a. Heinous crimeb. frustrated crimec. consummated crimed. attempted crime

21. Once criterion to assess the quality of a study is triangulation. This means.

a. accuracy and consistency of informationb. soundness of evidencec. use of multiple sources for conclusionsd. consistency and stability of evidence over time

22. It is thought to be the most recent method or system in providing nursing care

a. Case System methodb. primary nursing methodc. functional nursing methodd. team nursing method

23. A patient classification system where patients minimal theraphy and less frequent observation

a. minimal care (category 1)b. moderate care (category 2)c. maximum care (category 3)d. intensive care (category 4)

24. The nurse is to apply a dressing to a stage II pressure ulcer. Which of the following dressing is best?

a. Dry gauze dressingb. wet gauze dressingc. wet to dry dressingd. moisture vapor permeable dressing

25. The client has been placed in the trendelenburg position. The nurse knows

the effects of this position to the client include which of the following.

a. increase blood flow to the feetb. decrease blood pressurec. increase pressure on the diaphragmd. decrease intracranial pressure

26. A man who has been in an MVA is going into shock. Before placing the client in a modified trendelenburg position, the nurse should assess the client for:

a. long bone fractureb. air embolusc. head injuryd. thrombophlebitis

27. The nurse enter a room and finds a fire. Which is the best initial action?

a. Evacuate any people in the room, beginning with the most ambultory and ending with the least mobileb. activating the fire alarm or call the operator, depending on the institutions systemc. get a fire extinguisher and put out the fired. close all the windows and doors and turn off any oxygen or electricity appliance.

28. A member of the board of nursing may be removed and/or suspended on the following grounds except:

a. Toleration of irregularities in the licensure examinationb Violation of the Philippine Nursing Actc. Neglect of duty or incompetenced. Immoral unprofessional or dishonorable conduct

29. A tendency of the evaluator to inaacurately rate a group because of the belief that he/she has a very good cooperative group.

a. hawthorne effectb. sunflower effectc. horn's effectd. halo effect

30. A type of probability sampling where the researcher selects random samples, successively from a larger to smaller units by either simple or stratified methods.

a. Multi-stage samplingb. Systematic samplingc. purposive samplingd. snowballing sampling

31. The nurse is to open a sterile package from central supply. Which is the correct direction to open the first lap?

a. Toward the nurseb. Away from the nursec. To the nurse's left or right handd. It does not matter as long as the nurse touches only the outside edge

32. The nurse knows which of the following is the proper technique for medical asepsis?

a. gloving for all the client contactb. changing hospital linens weeklyc. using your hands to turn off the faucet after handwashingd. gowning to care for a 1 year old child w/ infections diarrhea

33. An adult ha a left, above the knee amputation two weeks ago. The nurse places him in a prone position tree times a day because

a. Prevents pressure ulcer on the sacrumb. helps the prosthesis to fit correctlyc. prevents flexion constracturesd. allow better blood flow to the heart

34. A woman is to have a pelvic exam. Which of the following should the nurse have the client do first?

a. Remove all her clothes and her socks and shoesb. go to the bahtroom and void saving a samplec. assume a lithotomy position on the exam tabled. assemble all the equipments needed for the examination

35. An adult is supine. Which of the ff. can the nurse to to prevent external rotation of the legs?

a. put a pillow under the clients lower legsb. place a pillow directly under the client kneec. use a trochanter rool alongside the client's upper thighsd. lower the client's legs so that they ae

Page 21: Document1

below hips.

36. The following are elements of felony except?

a. There must be an act of ommisionb. such act or omission must be done voluntarilyc. such act maybe done with or without intentd. such act or omission is punisable by law

37. To qualify as a dean of the college of nursing the applicants must have?

a. At least 5 years of teaching and supervisionb. At least 10 years of clinical practivec. At least 3 years of being a clinical coordinatord. At least 2 years management units.

38. A research design where there is manipulation of independent variable but no random or control group

a. longitudinal designb. quasi-experimentalc. experimentald. cross-sectional

39. Four clients have signaled with the call bells for the nurse. Who should you see first?

a. A client who needs to use the toiletb. A client who does not have his glasses or hearing aidec. A client who has just been given morphined. A client in a chair with a restraint vest on.

40. Which of the ff. should the nurse observe first?

a. A client who has just returned from the ORb. A client whose call light is not workingc. A client with Alzheimer's diseased. A clietn who is receiving a heating pad treatment

41. Which of the ff. should the nurse deal first?

a. A client who need her dressing changedb. A client who needs to be suctionedc. A client needs to be medicated for

postoperative paind. A client who is incontinent and needs to be cleaned

42. One of the major ethical princles in research is beneficence that includes:

a. Right to protection of protection from physical and psychological harmb. Right to full-disclosurec. Right to performance of sources goodd. Right to protection of participants form explotation.

43. He described humans as "wanting organism" that satisfly their basic needs in a specific sequence?

a. Frederick Herzbergb. Abraham Maslowc. BF Skinnerd. Victor Vroom

50. A client has been receiving an IV solution. What is an appropriate expected outcome for this client?

a. Monitor fluid intake and output every 4 hoursb. The client remains free of signs and symptoms of phelibitsc. edema and warmth over the IV sited. Excess fluid volumn evidenced by weight gain

51. This theorist of management developed the 14 principles of management based on his experience.

a. Mary Follettb. Max Weberc. Henry Fayold. Kurt Lewin

52. The nurse prepares to palpate a clients maxillary sinues. For this procedure, where should the nurse place the hands?

a. On the bridge of the noseb. below the eyebrowsc. below the cheekbonesd. over the temporal area

53. A client who receives general anesthesia returns from surgery. Postoperatively, which nursing diagnosis takes highest priority for this client?

a. Pain related to the surgeryb. Fluid volume deficit related to fluid and blood loss from surgeryc. Impared physical mobility related to surgeryd. Risk for aspiration related to anesthesia

54. After a client receives an IM injection, he complains of a burning pain in the injection site. Which nursing action whould be best to take at this time?

a. apply a cold compress to decrease swellingb. apply a warm compress to dilate the blood vesselsc. Massage the area to promote absorption of the drugd. Instruct the client to tighten his gluteal muscles to enhance absorption of the drug

55. These are mitigating circumstances that lessen the penalty, except:

a. Under 18 and over 70b. No intention to commit so gravec. There wa sufficent provocationd. in contempt of authority

56. The nurse is monitoring options of effectiveness of drug theraphy, when should the nurse obtain a measurement of the blood sample through level.

a. 1 hour before administering the next doseb. immediately before administering the next dosec. immediately after administering the next dosed. 30 mins. after administering the next dose

57. Which od the following options servers a framework for nursing education and clinical practice?

a. Scientific breakthroughb. technological advancesc. theoretical modelsd. medical practices

58. Nursing care for a client includes removing elastic stocking once per day. What is the rationale for this interventions?

a. TO increase blood flow to the heartb. to observe the lower extremities

Page 22: Document1

c. to allow the leg muscles to stretch and relaxd. To permit veins in the legs to fill with blood.

59. Hyperactive bowel sounds can result from all the following except.

a. Paralytic ileusb. hungerc. intestinal obstructiond. diarrhea

60. The following characteristics speak of a leader except:

a. Do the right thingb. focus on peoplec. develops visions and strategiesd. have a short-term views

61. This theory of management suggest that an employees work motivation is controlled by conditions in the external environment rather than by internal needs and desires.

a. vroom's expectancy theoryb. skinner's operant theoryc. adam's equity theoryd. white competence theory

62. In Lydia Hall's theory, which of the ff. represents "core"?

a. Nurturance and factors that are exclusive to nursingb. Therapeutic use of selfc. Focuses on nursing related to physicians orderd. All of the above.

63. A research that seeks to describe an existing problem situation and examine the underlying factors that contributes to the emergence of the problem:

a. Explanatory researchb. Quantitative researchc. Exploratory researchd. Quantitative research

64. During blood transfusion, the patient manifest tacycardia, distended neck vein and increase CVP reading, the nurse should

a. Obtain vital signsb. Call the physician

c. Stop the infusiond. Decrease the rate of infusion.

65. A nurse who is maintainin a private clinic in the community and renders service on materials and child health amont the neighborhood for a fee is

a. primary care nurseb. an independent nurse practionerc. nurse-midwifed. a nurse specialist

66. A client comes to the clinic for a diagnostic allergy testing. Why is intradermal injections used for such tsting.

a. intradermal injection is less painfulb. intradermal drugs are easier to administerc. intradermal drug diffuse more rapidlyd. intradermal drugs diffuse more slowly

67. A client is placed in isolation. Client isolation techniques attempt to break the chain of infection by interfering with the

a. Agentb. susceptible hostc. transmission moded. portal of entry68. A research hypothesis that proposes that there is association or significant relationship between variables is called.a. null hypothesisb. directional hypothesisc. alternative hypothesisd. non-directional hypothesis69. Which of the ff. laboratory test results is themost impt. indicator of malnutrition in a client with a wound?a. serum potassium levelb. albumin levelc. lymphocyte countd. differential count70. Which detail of a clients drug theraphy is the nurse legally responsible for documenting?a. Peak concentration time of drugb. Safe ranges of the drugc. Clients socioeconomic datad. client's reaction to the drug71. Which of the ff. statements about these nursing practice acts are true?a. These acts are not effective in the supervision or revocation of licensure of erring nurses

b. These acts have been authorized by the accredited national nurses organizationc. These acts usually define the scope of nursing practiced. These acts describe the diffences between a registered nurse and a practice nurse72. The following are true on a manager except?a. Are interested in efficiencyb. emphasize tactics, structure and systemc. motivate people to comply with standardsd. use person to person influence73. The nurse is assessing tactile fremitus in a client with pneumonia. For this examination, the nurse hould use the:a. fingertipsb. ulnar surface of the handc. dorsal surface of the handd. finger pads74. The ff. activities belong to the design phase in the research process, except?a. developing a time tableb. selecting the study designc. determining sampling pland. conducting a pilot study75. When inspecting a client's skin, the nurse finds a vesicle on the client's arm. Which description appliest to a vesicle?a. flat, nonpalpable and coloredb. solid, elevated and cicumscribedc. circumscribed, elevated, and filled with serous fluidd. elevated, pus-filled and circumscribed76. Which nursing action is essential when providing continuous enteral feeding?a. elevating the head of the bedb. positioning the client on the left sidec. warming the formula before administering itd. hanging a full day's worth of formula at one time77. After initial assessment of a client, findings reveals a body temp of 38.5 C, Consequently the clients blood pressure can be expected toa. Fluctuatesb. Remain the Samec. Increased. Decrease78. The nurse assesses the skin turgor of an elderly client. When evaluating skin turgon, the nurse should remember that:

a. Over hydration causes the skin to tentb. Dehydration causes the skin to appear edematous and spongy

Page 23: Document1

c. Inelastic turgor is a normal part of agingd. Normal skin turgon is moist and boggy.

79. Which of the ff. does not govern the practice of nursing?

a. RA 7164b. RA 9173c. BON Resolution for Code of Ethicsd. Board Resolution Scope of Nursing Practice

80. A client with sever chest pain is brought to the emergency department. He tells the nurse, "I just have a little indigestion." Which of the following mechanisms is the client exhibiting?

a. Denialb. Anxietyc. Repressiond. Confusion

81. These are "in-between factors" present in research environment that threaten the internal and external validity of a research study.

a. Dependent variableb. Independent variablesc. Intervening variablesd. Criterion variable

82. The ff. are motivational factors according to Herzberg, except:

a. Challenging aspects of the work itselfb. Added responsibilityc. Oppurtunities for personal growthd. Quality of supervision

83. The nurse measures the client's temperature at 102 F. What is the equivalent centigrade temp?

a. 39.1 Cb. 47.2 Cc. 38.9 Cd. 40.1 C

84. A client is placed on a low-sodium (500mg/day) diet. Which client statement indicates that the nurse's nutrition teaching plan has been effective?

a. "I can eat a ham-and-cheese sandwich w/ potato chips for lunch"b. "I choose broiled chiken whith a baked potato for dinner"

c. "I choose a tossed salad w/ sardines and oil and vinegar dressing for lunch"d. "Im glad i can still have chicken bouillon soup"

85. Why should an infant be quiet and seated upright when the nurse assess his fontaanels?

a. The mother will have less trouble holding a quiet upright infantb. Lying down can cause the fontanels to recede, making assessment more difficult.c. The infant can breath more easily when sitting upd. Lying down and crying can cause the fontanels to buldge

86. Communicating and using the research findings is under what phase of the research process?

a. Implementation phaseb. Dissemination phasec. Analysis and interpretation phased. Design phase

87. To give a Z-tract injection, the nuse measures the correct medication dose and draws a small amount of air into the syringe. What is the rationale for this action?

a. Adding air decreases pain caused by the injectionb. Adding air prevents the drug from flowing back into the needle tractc. Adding air prevens the solution from entering a blood vesseld. Adding air ensures that client receives the entire dose.

88. Which of the ff. is true of Care System method?

a. Autonomy and authority for the care of four to siz patients on a 24-h basisb. Assigning nurses to give total nursing care to a single patientc. Divison of labor which nurses are assigned to perform secific tasksd. Assigning a team to give total care to a selected group of clients

89. Which of the ff. best describes the action of the nurse who documents her nursing diagnosis?

a. She documents and charts it whenever

necessaryb. She can be accused of malpracticec. She does it reguarly as an impt. responsibilityd. She starts it only when the pt. is acutely ill

90. To help assess a clients cerabral function, the nurse should ask:

a. Have you noticed a change in your memory?b. Have you noticed a chnge in your muscle strength?c. Have you had any coodination problem?d. Have you had any problems with your eyes?

91. The nurse is evaulating a postoperative client for infection. Which sign of sympton would be most indicative for infection?

a. The presence of an indwelling urinary catheterb. Rectal temperature of 100Fc. Red, warm , tender incisiond. WVC count of 8,000/UI

92. A patient classification system where patients need close attention and complete care in most activities and requires frequent and complex treatments and medications

a. Minimal Care (category 1)b. Moderate Care (category 2)c. Maximum Care (category 3)d. Intensive Care (category 4)

93. What nursing diagnosis is commonly used to develop a plan of care for managing client's drug regimen?

a. Noncomplienceb. Altered coping mechanismc. Anxietyd. Altered home maintenance

94. Which of the ff. provides that nurses be members of the national nurse organization?

a. RA 877b. 1987 Code of Ethics approved by the house of delegates PNAc. Board Resolution No 1955 promulgated by the Board of Nursingd. RA 7164

Page 24: Document1

95. The ff. are good sources of research problems except?

a. Literatureb. Clininal experiencec. Daily news reportsd. existing theories

96. What is the common goal of discharge planning in all care settings?

a. prolonging hospitalization until the client can function independentlyb. Teaching client how to perform self-care activitiesc. Providing the financial resources needed to ensure proper cared. Preventing the need for medical follow-up care.

97. The physician prescribe as IV infucion of 1000 ml D5W every 12 hours for a client. The IV setup delivers 15 gtt/ml. Approx. how many drops should be administered each minute?

a. 18b. 21c. 24d. 27

98. Which of the following provides the material and human resources and building the structure to care out the activities of the organization?

a. Planningb. Organizingc. Commandd. Control

99. A client is scheduled to receive a blood transfusion. During the transfusion, the nurse should observer for which sign or symptoms of a transfusion reaction?

a. Dizzinessb. Chillsc. Hypothermiad. Hyperreflexia

100. The client has an abdominal hysterectomy, 10 hours ago. Which position should the nurse teach the client to avoid?

a. High Fowlersb. lateral recumbent

c. Supined. Side lying

Page 25: Document1

SITUATION : Arthur, A registered nurse, witnessed an old woman hit by a motorcycle while crossing a train railway. The old woman fell at the railway. Arthur rushed at the scene.

1. As a registered nurse, Arthur knew that the first thing that he will do at the scene is

A. Stay with the person, Encourage her to remain still and Immobilize the leg whileWhile waiting for the ambulance.B. Leave the person for a few moments to call for help.C. Reduce the fracture manually.D. Move the person to a safer place.

2. Arthur suspects a hip fracture when he noticed that the old woman’s leg is

A. Lengthened, Abducted and Internally Rotated.B. Shortened, Abducted and Externally Rotated.C. Shortened, Adducted and Internally Rotated.D. Shortened, Adducted and Externally Rotated.

3. The old woman complains of pain. John noticed that the knee is reddened, warm to touch and swollen. John interprets that this signs and symptoms are likely related to

A. InfectionB. ThrombophlebitisC. InflammationD. Degenerative disease

4. The old woman told John that she has osteoporosis; Arthur knew that all of the following factors would contribute to osteoporosis except

A. HypothyroidismB. End stage renal diseaseC. Cushing’s DiseaseD. Taking Furosemide and Phenytoin.

5. Martha, The old woman was now Immobilized and brought to the emergency room. The X-ray shows a fractured femur and pelvis. The ER Nurse would carefully monitor Martha for which of the following sign and symptoms?

A. Tachycardia and HypotensionB. Fever and BradycardiaC. Bradycardia and HypertensionD. Fever and Hypertension

SITUATION: Mr. D. Rojas, An obese 35 year old MS Professor of OLFU Lagro is admitted due to pain in his weight bearing joint. The diagnosis was Osteoarthritis.

6. As a nurse, you instructed Mr. Rojas how to use a cane. Mr. Rojas has a weakness on his right leg due to self immobilization and guarding. You plan to teach Mr. Rojas to hold the cane

A. On his left hand, because his right side is weak.B. On his left hand, because of reciprocal motion.C. On his right hand, to support the right leg.D. On his right hand, because only his right leg is weak.

7. You also told Mr. Rojas to hold the cane

A. 1 Inches in front of the foot.B. 3 Inches at the lateral side of the foot.c. 6 Inches at the lateral side of the foot.D. 12 Inches at the lateral side of the foot.

8. Mr. Rojas was discharged and 6 months later, he came back to the emergency room of the hospital because he suffered a mild stroke. The right side of the brain was affected. At the rehabilitative phase of your nursing care, you observe Mr. Rojas use a cane and you intervene if you see him

A. Moves the cane when the right leg is moved.B. Leans on the cane when the right leg swings through.C. keeps the cane 6 Inches out to the side of the right foot.D. Holds the cane on the right side.

SITUATION: Alfred, a 40 year old construction worker developed cough, night sweats and fever. He was brought to the nursing unit for diagnostic studies. He told the nurse he did not receive a BCG vaccine during childhood

9. The nurse performs a Mantoux Test. The nurse knows that Mantoux Test is also known as

A. PPDB. PDPC. PDDD. DPP

10. The nurse would inject the solution in what route?

A. IMB. IVC. IDD. SC

11. The nurse notes that a positive result for Alfred is

A. 5 mm whealB. 5 mm IndurationC. 10 mm WhealD. 10 mm Induration

12. The nurse told Alfred to come back after

A. a weekB. 48 hoursC. 1 dayD. 4 days

13. Mang Alfred returns after the Mantoux Test. The test result read POSITIVE. What should be the nurse’s next action?

A. Call the PhysicianB. Notify the radiology dept. for CXR evaluationC. Isolate the patientD. Order for a sputum exam

14. Why is Mantoux test not routinely done in the Philippines?

A. It requires a highly skilled nurse to perform a Mantoux testB. The sputum culture is the gold standard of PTB Diagnosis and it will definitively determine the extent of the cavitary lesionsC. Chest X Ray Can diagnose the specific microorganism responsible for the lesionsD. Almost all Filipinos will test positive for Mantoux Test

15. Mang Alfred is now a new TB patient with an active disease. What is his category according to the DOH?

A. I

Page 26: Document1

B. IIC. IIID. IV

16. How long is the duration of the maintenance phase of his treatment?

A. 2 monthsB. 3 monthsC. 4 monthsD. 5 months

17. Which of the following drugs is UNLIKELY given to Mang Alfred during the maintenance phase?

A. RifampicinB. IsoniazidC. EthambutolD. Pyridoxine

18. According to the DOH, the most hazardous period for development of clinical disease is during the first

A. 6-12 months afterB. 3-6 months afterC. 1-2 months afterD. 2-4 weeks after

19. This is the name of the program of the DOH to control TB in the country

A. DOTSB. National Tuberculosis Control ProgramC. Short Coursed ChemotherapyD. Expanded Program for Immunization

20. Susceptibility for the disease [ TB ] is increased markedly in those with the following condition except

A. 23 Year old athlete with diabetes insipidusB. 23 Year old athlete taking long term Decadron therapy and anabolic steroidsC. 23 Year old athlete taking illegal drugs and abusing substancesD. Undernourished and Underweight individual who undergone gastrectomy

21. Direct sputum examination and Chest X ray of TB symptomatic is in what level of prevention?

A. PrimaryB. SecondaryC. TertiaryD. Quarterly

SITUATION: Michiel, A male patient diagnosed with colon cancer was newly put in colostomy.

22. Michiel shows the BEST adaptation with the new colostomy if he shows which of the following?

A. Look at the ostomy siteB. Participate with the nurse in his daily ostomy careC. Ask for leaflets and contact numbers of ostomy support groupsD. Talk about his ostomy openly to the nurse and friends

23. The nurse plans to teach Michiel about colostomy irrigation. As the nurse prepares the materials needed, which of the following item indicates that the nurse needs further instruction?

A. Plain NSS / Normal SalineB. K-Y JellyC. Tap waterD. Irrigation sleeve

24. The nurse should insert the colostomy tube for irrigation at approximately

A. 1-2 inchesB. 3-4 inchesC. 6-8 inchesD. 12-18 inches

25. The maximum height of irrigation solution for colostomy is

A. 5 inchesB. 12 inchesC. 18 inchesD. 24 inches

26. Which of the following behavior of the client indicates the best initial step in learning to care for his colostomy?

A. Ask to defer colostomy care to another individualB. Promises he will begin to listen the next dayC. Agrees to look at the colostomyD. States that colostomy care is the function of the nurse while he is in the hospital

27. While irrigating the client’s colostomy, Michiel suddenly complains of severe

cramping. Initially, the nurse would

A. Stop the irrigation by clamping the tubeB. Slow down the irrigationC. Tell the client that cramping will subside and is normalD. Notify the physician

28. The next day, the nurse will assess Michiel’s stoma. The nurse noticed that a prolapsed stoma is evident if she sees which of the following?

A. A sunken and hidden stomaB. A dusky and bluish stomaC. A narrow and flattened stomaD. Protruding stoma with swollen appearance

29. Michiel asked the nurse, what foods will help lessen the odor of his colostomy. The nurse best response would be

A. Eat eggsB. Eat cucumbersC. Eat beet greens and parsleyD. Eat broccoli and spinach

30. The nurse will start to teach Michiel about the techniques for colostomy irrigation. Which of the following should be included in the nurse’s teaching plan?

A. Use 500 ml to 1,000 ml NSSB. Suspend the irrigant 45 cm above the stomaC. Insert the cone 4 cm in the stomaD. If cramping occurs, slow the irrigation

31. The nurse knew that the normal color of Michiel’s stoma should be

A. Brick RedB. GrayC. BlueD. Pale Pink

SITUATION: James, A 27 basketball player sustained inhalation burn that required him to have tracheostomy due to massive upper airway edema.

32. Wilma, His sister and a nurse is suctioning the tracheostomy tube of James. Which of the following, if made by Wilma indicates that she is committing an error?

A. Hyperventilating James with 100%

Page 27: Document1

oxygen before and after suctioningB. Instilling 3 to 5 ml normal saline to loosen up secretionC. Applying suction during catheter withdrawalD. Suction the client every hour

33. What size of suction catheter would Wilma use for James, who is 6 feet 5 inches in height and weighing approximately 145 lbs?

A. Fr. 5B. Fr. 10C. Fr. 12D. Fr. 18

34. Wilma is using a portable suction unit at home, What is the amount of suction required by James using this unit?

A. 2-5 mmHgB. 5-10 mmHgC. 10-15 mmHgD. 20-25 mmHg

35. If a Wall unit is used, What should be the suctioning pressure required by James?

A. 50-95 mmHgB. 95-110 mmHgC. 100-120 mmHgD. 155-175 mmHg

36. Wilma was shocked to see that the Tracheostomy was dislodged. Both the inner and outer cannulas was removed and left hanging on James’ neck. What are the 2 equipment’s at james’ bedside that could help Wilma deal with this situation?

A. New set of tracheostomy tubes and Oxygen tankB. Theophylline and EpinephrineC. Obturator and Kelly clampD. Sterile saline dressing

37. Which of the following method if used by Wilma will best assure that the tracheostomy ties are not too tightly placed?

A. Wilma places 2 fingers between the tie and neckB. The tracheotomy can be pulled slightly away from the neckC. James’ neck veins are not engorgedD. Wilma measures the tie from the nose

to the tip of the earlobe and to the xiphoid process.

38. Wilma knew that James have an adequate respiratory condition if she notices that

A. James’ respiratory rate is 18B. James’ Oxygen saturation is 91%C. There are frank blood suction from the tubeD. There are moderate amount of tracheobronchial secretions

39. Wilma knew that the maximum time when suctioning James is

A. 10 secondsB. 20 secondsC. 30 secondsD. 45 seconds

SITUATION : Juan Miguel Lopez Zobel Ayala de Batumbakal was diagnosed with Acute Close Angle Glaucoma. He is being seen by Nurse Jet.

40. What specific manifestation would nurse Jet see in Acute close angle glaucoma that she would not see in an open angle glaucoma?

A. Loss of peripheral visionB. Irreversible vision lossC. There is an increase in IOPD. Pain

41. Nurse jet knew that Acute close angle glaucoma is caused by

A. Sudden blockage of the anterior angle by the base of the irisB. Obstruction in trabecular meshworkC. Gradual increase of IOPD. An abrupt rise in IOP from 8 to 15 mmHg

42. Nurse jet performed a TONOMETRY test to Mr. Batumbakal. What does this test measures

A. It measures the peripheral vision remaining on the clientB. Measures the Intra Ocular PressureC. Measures the Client’s Visual AcuityD. Determines the Tone of the eye in response to the sudden increase in IOP.

43. The Nurse notices that Mr. Batumbakal

cannot anymore determine RED from BLUE. The nurse knew that which part of the eye is affected by this change?

A. IRISB. PUPILc. RODS [RETINA]D. CONES [RETINA]

44. Nurse Jet knows that Aqueous Humor is produce where?

A. In the sub arachnoid space of the meningesB. In the Lateral ventriclesC. In the ChoroidsD. In the Ciliary Body

45. Nurse Jet knows that the normal IOP is

A. 8-21 mmHgB. 2-7 mmHgc. 31-35 mmHgD. 15-30 mmHg

46. Nurse Jet wants to measure Mr. Batumbakal’s CN II Function. What test would Nurse Jet implement to measure CN II’s Acuity?

A. Slit lampB. Snellen’s ChartC. Wood’s lightD. Gonioscopy

47. The Doctor orders pilocarpine. Nurse jet knows that the action of this drug is to

A. Contract the Ciliary muscleB. Relax the Ciliary muscleC. Dilate the pupilsD. Decrease production of Aqueous Humor

48. The doctor orders timolol [timoptic]. Nurse jet knows that the action of this drug is

A. Reduce production of CSFB. Reduce production of Aquesous HumorC. Constrict the pupilD. Relaxes the Ciliary muscle

49. When caring for Mr. Batumbakal, Jet teaches the client to avoid

A. Watching large screen TVsB. Bending at the waistC. Reading booksD. Going out in the sun

Page 28: Document1

50. Mr. Batumbakal has undergone eye angiography using an Intravenous dye and fluoroscopy. What activity is contraindicated immediately after procedure?

A. Reading newsprintB. Lying downC. Watching TVD. Listening to the music

51. If Mr. Batumbakal is receiving pilocarpine, what drug should always be available in any case systemic toxicity occurs?

A. Atropine SulfateB. Pindolol [Visken]C. Naloxone Hydrochloride [Narcan]D. Mesoridazine Besylate [Serentil]

SITUATION : Wide knowledge about the human ear, it’s parts and it’s functions will help a nurse assess and analyze changes in the adult client’s health.

52. Nurse Anna is doing a caloric testing to his patient, Aida, a 55 year old university professor who recently went into coma after being mauled by her disgruntled 3rd year nursing students whom she gave a failing mark. After instilling a warm water in the ear, Anna noticed a rotary nystagmus towards the irrigated ear. What does this means?

A. Indicates a CN VIII DysfunctionB. AbnormalC. NormalD. Inconclusive

53. Ear drops are prescribed to an infant, The most appropriate method to administer the ear drops is

A. Pull the pinna up and back and direct the solution towards the eardrumB. Pull the pinna down and back and direct the solution onto the wall of the canalC. Pull the pinna down and back and direct the solution towards the eardrumD. Pull the pinna up and back and direct the solution onto the wall of the canal

54. Nurse Jenny is developing a plan of care for a patient with Menieres disease. What is the priority nursing intervention in

the plan of care for this particular patient?

A. Air, Breathing, CirculationB. Love and BelongingnessC. Food, Diet and NutritionD. Safety

55. After mastoidectomy, Nurse John should be aware that the cranial nerve that is usually damage after this procedure is

A. CN IB. CN IIC. CN VIID. CN VI

56. The physician orders the following for the client with Menieres disease. Which of the following should the nurse question?

A. Dipenhydramine [Benadryl]B. Atropine sulfateC. Out of bed activities and ambulationD. Diazepam [Valium]

57. Nurse Anna is giving dietary instruction to a client with Menieres disease. Which statement if made by the client indicates that the teaching has been successful?

A. I will try to eat foods that are low in sodium and limit my fluid intakeB. I must drink atleast 3,000 ml of fluids per dayC. I will try to follow a 50% carbohydrate, 30% fat and 20% protein dietD. I will not eat turnips, red meat and raddish

58. Peachy was rushed by his father, Steven into the hospital admission. Peachy is complaining of something buzzing into her ears. Nurse Joemar assessed peachy and found out It was an insect. What should be the first thing that Nurse Joemar should try to remove the insect out from peachy’s ear?

A. Use a flashlight to coax the insect out of peachy’s earB. Instill an antibiotic ear dropsC. Irrigate the earD. Pick out the insect using a sterile clean forceps

59. Following an ear surgery, which statement if heard by Nurse Oca from the patient indicates a correct understanding

of the post operative instructions?

A. Activities are resumed within 5 daysB. I will make sure that I will clean my hair and face to prevent infectionC. I will use straw for drinkingD. I should avoid air travel for a while

60. Nurse Oca will do a caloric testing to a client who sustained a blunt injury in the head. He instilled a cold water in the client’s right ear and he noticed that nystagmus occurred towards the left ear. What does this finding indicates?

A. Indicating a Cranial Nerve VIII DysfunctionB. The test should be repeated again because the result is vagueC. This is Grossly abnormal and should be reported to the neurosurgeonD. This indicates an intact and working vestibular branch of CN VIII

61. A client with Cataract is about to undergo surgery. Nurse Oca is preparing plan of care. Which of the following nursing diagnosis is most appropriate to address the long term need of this type of patient?

A. Anxiety R/T to the operation and its outcomeB. Sensory perceptual alteration R/T Lens extraction and replacementC. Knowledge deficit R/T the pre operative and post operative self careD. Body Image disturbance R/T the eye packing after surgery

62. Nurse Joseph is performing a WEBERS TEST. He placed the tuning fork in the patients forehead after tapping it onto his knee. The client states that the fork is louder in the LEFT EAR. Which of the following is a correct conclusion for nurse Josph to make?

A. He might have a sensory hearing loss in the left earB. Conductive hearing loss is possible in the right earC. He might have a sensory hearing loss in the right hear, and/or a conductive hearing loss in the left ear.D. He might have a conductive hearing loss in the right ear, and/or a sensory hearing loss in the left ear.

Page 29: Document1

63. Aling myrna has Menieres disease. What typical dietary prescription would nurse Oca expect the doctor to prescribe?

A. A low sodium , high fluid intakeB. A high calorie, high protein dietary intakeC. low fat, low sodium and high calorie intakeD. low sodium and restricted fluid intake

SITUATION : [ From DEC 1991 NLE ] A 45 year old male construction worker was admitted to a tertiary hospital for incessant vomiting. Assessment disclosed: weak rapid pulse, acute weight loss of .5kg, furrows in his tongue, slow flattening of the skin was noted when the nurse released her pinch.

Temperature: 35.8 C , BUN Creatinine ratio : 10 : 1, He also complains for postural hypotension. There was no infection.

64. Which of the following is the appropriate nursing diagnosis?

A. Fluid volume deficit R/T furrow tongueB. Fluid volume deficit R/T uncontrolled vomitingC. Dehydration R/T subnormal body temperatureD. Dehydration R/T incessant vomiting

65. Approximately how much fluid is lost in acute weight loss of .5kg?

A. 50 mlB. 750 mlC. 500 mlD. 75 ml

66. Postural Hypotension is

A. A drop in systolic pressure less than 10 mmHg when patient changes position from lying to sitting.B. A drop in systolic pressure greater than 10 mmHg when patient changes position from lying to sittingC. A drop in diastolic pressure less than 10 mmHg when patient changes position from lying to sittingD. A drop in diastolic pressure greater than 10 mmHg when patient changes position from lying to sitting

67. Which of the following measures will

not help correct the patient’s condition

A. Offer large amount of oral fluid intake to replace fluid lostB. Give enteral or parenteral fluidC. Frequent oral careD. Give small volumes of fluid at frequent interval

68. After nursing intervention, you will expect the patient to have

1. Maintain body temperature at 36.5 C2. Exhibit return of BP and Pulse to normal3. Manifest normal skin turgor of skin and tongue4. Drinks fluids as prescribed

A. 1,3B. 2,4C. 1,3,4D. 2,3,4

SITUATION: A 65 year old woman was admitted for Parkinson’s Disease. The charge nurse is going to make an initial assessment.

69. Which of the following is a characteristic of a patient with advanced Parkinson’s disease?

A. Disturbed visionB. ForgetfulnessC. Mask like facial expressionD. Muscle atrophy

70. The onset of Parkinson’s disease is between 50-60 years old. This disorder is caused by

A. Injurious chemical substancesB. Hereditary factorsC. Death of brain cells due to old ageD. Impairment of dopamine producing cells in the brain

71. The patient was prescribed with levodopa. What is the action of this drug?

A. Increase dopamine availabilityB. Activates dopaminergic receptors in the basal gangliaC. Decrease acetylcholine availabilityD. Release dopamine and other catecholamine from neurological storage sites

72. You are discussing with the dietician

what food to avoid with patients taking levodopa?

A. Vitamin C rich foodB. Vitamin E rich foodC. Thiamine rich foodD. Vitamin B6 rich food

73. One day, the patient complained of difficulty in walking. Your response would be

A. You will need a cane for supportB. Walk erect with eyes on horizonC. I’ll get you a wheelchairD. Don’t force yourself to walk

SITUATION: Mr. Dela Isla, a client with early Dementia exhibits thought process disturbances.

74. The nurse will assess a loss of ability in which of the following areas?

A. BalanceB. JudgmentC. SpeechD. Endurance

75. Mr. Dela Isla said he cannot comprehend what the nurse was saying. He suffers from:

A. InsomniaB. AphraxiaC. AgnosiaD. Aphasia

76. The nurse is aware that in communicating with an elderly client, the nurse will

A. Lean and shout at the ear of the clientB. Open mouth wide while talking to the clientC. Use a low-pitched voiceD. Use a medium-pitched voice

77. As the nurse talks to the daughter of Mr. Dela Isla, which of the following statement of the daughter will require the nurse to give further teaching?

A. I know the hallucinations are parts of the diseaseB. I told her she is wrong and I explained to her what is rightC. I help her do some tasks he cannot do for himself

Page 30: Document1

D. Ill turn off the TV when we go to another room

78. Which of the following is most important discharge teaching for Mr. Dela Isla

A. Emergency NumbersB. Drug ComplianceC. Relaxation techniqueD. Dietary prescription

SITUATION : Knowledge of the drug PROPANTHELINE BROMIDE [Probanthine] Is necessary in treatment of various disorders.

79. What is the action of this drug?

A. Increases glandular secretion for clients affected with cystic fibrosisB. Dissolve blockage of the urinary tract due to obstruction of cystine stonesC. Reduces secretion of the glandular organ of the bodyD. Stimulate peristalsis for treatment of constipation and obstruction

80. What should the nurse caution the client when using this medication

A. Avoid hazardous activities like driving, operating machineries etc.B. Take the drug on empty stomachC. Take with a full glass of water in treatment of Ulcerative colitisD. I must take double dose if I missed the previous dose

81. Which of the following drugs are not compatible when taking Probanthine?

A. CaffeineB. NSAIDC. AcetaminophenD. Alcohol

82. What should the nurse tell clients when taking Probanthine?

A. Avoid hot weathers to prevent heat strokesB. Never swim on a chlorinated poolC. Make sure you limit your fluid intake to 1L a dayD. Avoid cold weathers to prevent hypothermia

83. Which of the following disease would

Probanthine exert the much needed action for control or treatment of the disorder?

A. Urinary retentionB. Peptic Ulcer DiseaseC. Ulcerative ColitisD. Glaucoma

SITUATION : Mr. Franco, 70 years old, suddenly could not lift his spoons nor speak at breakfast. He was rushed to the hospital unconscious. His diagnosis was CVA.

84. Which of the following is the most important assessment during the acute stage of an unconscious patient like Mr. Franco?

A. Level of awareness and response to painB. Papillary reflexes and response to sensory stimuliC. Coherence and sense of hearingD. Patency of airway and adequacy of respiration

85. Considering Mr. Franco’s conditions, which of the following is most important to include in preparing Franco’s bedside equipment?

A. Hand bell and extra bed linenB. Sandbag and trochanter rollsC. Footboard and splintD. Suction machine and gloves

86. What is the rationale for giving Mr. Franco frequent mouth care?

A. He will be thirsty considering that he is doesn’t drink enough fluidsB. To remove dried blood when tongue is bitten during a seizureC. The tactile stimulation during mouth care will hasten return to consciousnessD. Mouth breathing is used by comatose patient and it’ll cause oral mucosa dying and cracking.

87. One of the complications of prolonged bed rest is decubitus ulcer. Which of the following can best prevent its occurrence?

A. Massage reddened areas with lotion or oilsB. Turn frequently every 2 hoursC. Use special water mattressD. Keep skin clean and dry

88. If Mr. Franco’s Right side is weak, What should be the most accurate analysis by the nurse?

A. Expressive aphasia is prominent on clients with right sided weaknessB. The affected lobe in the patient is the Right lobeC. The client will have problems in judging distance and proprioceptionD. Clients orientation to time and space will be much affected

SITUATION : a 20 year old college student was rushed to the ER of PGH after he fainted during their ROTC drill. Complained of severe right iliac pain. Upon palpation of his abdomen, Ernie jerks even on slight pressure. Blood test was ordered. Diagnosis is acute appendicitis.

89. Which result of the lab test will be significant to the diagnosis?

A. RBC : 4.5 TO 5 Million / cu. mm.B. Hgb : 13 to 14 gm/dl.C. Platelets : 250,000 to 500,000 cu.mm.D. WBC : 12,000 to 13,000/cu.mm

90. Stat appendectomy was indicated. Pre op care would include all of the following except?

A. Consent signed by the fatherB. Enema STATC. Skin prep of the area including the pubisD. Remove the jewelries

91. Pre-anesthetic med of Demerol and atrophine sulfate were ordered to :

A. Allay anxiety and apprehensionB. Reduce painC. Prevent vomitingD. Relax abdominal muscle

92. Common anesthesia for appendectomy is

A. SpinalB. GeneralC. CaudalD. Hypnosis

93. Post op care for appendectomy include the following except

A. Early ambulationB. Diet as tolerated after fully conscious

Page 31: Document1

C. Nasogastric tube connect to suctionD. Deep breathing and leg exercise

94. Peritonitis may occur in ruptured appendix and may cause serious problems which are

1. Hypovolemia, electrolyte imbalance2. Elevated temperature, weakness and diaphoresis3. Nausea and vomiting, rigidity of the abdominal wall4. Pallor and eventually shock

A. 1 and 2B. 2 and 3C. 1,2,3D. All of the above

95. If after surgery the patient’s abdomen becomes distended and no bowel sounds appreciated, what would be the most suspected complication?

A. IntussusceptionB. Paralytic IleusC. HemorrhageD. Ruptured colon

96. NGT was connected to suction. In caring for the patient with NGT, the nurse must

A. Irrigate the tube with saline as orderedB. Use sterile technique in irrigating the tubeC. advance the tube every hour to avoid kinksD. Offer some ice chips to wet lips

97. When do you think the NGT tube be removed?

A. When patient requests for itB. Abdomen is soft and patient asks for waterC. Abdomen is soft and flatus has been expelledD. B and C only

Situation: Amanda is suffering from chronic arteriosclerosis Brain syndrome she fell while getting out of the bed one morning and was brought to the hospital, and she was diagnosed to have cerebrovascular thrombosis thus transferred to a nursing home.

98. What do you call a STROKE that

manifests a bizarre behavior?

A. Inorganic StrokeB. Inorganic PsychosesC. Organic StrokeD. Organic Psychoses

99. The main difference between chronic and organic brain syndrome is that the former

A. Occurs suddenly and reversibleB. Is progressive and reversibleC. tends to be progressive and irreversibleD. Occurs suddenly and irreversible

100. Which behavior results from organic psychoses?

A. Memory deficitB. DisorientationC. Impaired JudgementD. Inappropriate affect

Page 32: Document1

Situation I -- Nurse Caria is assigned in the emergency unit meeting. Varied opportunities that developed her nursing skills.

1. A 17-year old is admitted following an automobile accident He is very anxious, dyspneic, and in severe pain. The left chest wall moves in during inspiration and balloons out when he exhales. The nurse understands these symptoms are most suggestive of:

a. Hemothoraxb. Flail chestc. Atelectasisd. Pleural effusion

2. A young man is admitted with a flail chest following a car accident. He is intubated with an endotracheal tube and is placed on a mechanical ventilator (control mode, positive pressure). Which physical finding alerts the nurse to an additional problem in respiratory function?

a. Dullness to percussion in the third to 5th intercostals space, midclavicular lineb. Decreased paradoxical motionc. Louder breath sounds on the right chestd. pH of 7.36 In arterial blood gases

3. The nurse is caring for a client who has just had a chest tube attached to a water seal drainage system (Pleur-evac). To ensure that the system functions effectively the nurse should:

a. Observe for intermittent bubbling in the water seal chamberb. Flush the test tube with 30 to 60 ml of NSS 4 to 6 hoursc. Maintain the client in an extreme lateral positiond. Strip the chest tubes in the direction of the client

4. The nurse enter the room of a client who has a chest tube attached to a water seal drainage system and notices the chest tube is dislodge from the chest. The most appropriate nursing intervention is to:

a. Notify the physicianb. Insert a new chest tubec. Cover the insertion site with new

petroleum gauzed. Instruct the client to breath deeply until the help arrives

5. A 71-year old is admitted to the hospital with congestive heart failure. She has shortness of breath and a +3 - 4 peripheral edema. The care plan to reduce the client's edema should include nursing strategies for:

a. Establishing limits on activityb. Fostering a relaxed environmentc. Identifying goals for self cared. Restricting IV fluids

Situation 2 - Oxygen is the most vital physiologic need for survival.

6. Mr. Sison, 65 years old has been smoking since he was 11 years old. He has long history of emphysema. Mr. Sison is admitted to the hospital because of a respiratory infection, which has not improved with outpatient therapy. Which finding would the nurse expect to observe during Mr. Sison's nursing assessment?

a. Electrocardiogram changesb. Increased anterior-posterior chest diameterc. Slow labored respiratory patternd. Weight-Height relationship indicating obesity

7. Mr. Sison is ordered oxygen via nasal prongs. The nurse administering the oxygen via the low-flow system recognizes that this method of delivery:

a. Mixes room air with oxygenb. Delivers a precise concentration of oxygenc. Requires humidity during deliveryd. Is less traumatic to the respirator tract

8. Which statement by Mr. Sison indicates that client teaching regarding oxygen therapy has been effective?

a. "I was feeling fine so I removed my nasal prongs."b. "I've increased my fluids to six glasses of water daily."c. "Don’t forget to come back quickly when you get me out of the bed; I don't want to be without my oxygen for too long." d. "My family was angry when I told them they could not smoke in my room."

9. Supplemental low-flow oxygen therapy is prescribed for a man with emphysema. Which is the most essential for the nurse to initiate?

a. Anticipate the need for humidification b. Notify the physician that this order is contraindicated c. Place client in high Fowler's position d. Schedule nursing care to allow frequent observations of the client

Situation 3 - Mr. Silverio, 56 years old, has had significant problem with alcohol abuse for the past 15 years. His wife brings him to the emergency department because he is increasingly confused and is coughing blood. His medical diagnosis is cirrhosis of the liver. He has ascites and esophageal varices.

10. Assessment of Mr. Silverio would reveal all of the following, except:

a. Bulging flanksb. Protruding umbilicusc. Abdominal distensiond. Bluish discoloration of the umbilicus

11. Which laboratory value would the nurse expect to find in a client as a result of liver failure?

a. Decreased serum creatinine b. Decreased sodium c. Increased ammonia d. Restricted sodium

12. The major dietary treatment for ascites calls for:

a. High proteinb. Increased potassiumc. Restricted fluidsd. Restricted sodium

13. A Sengstaken-BIakemore tube is inserted in an effort to stop the bleeding. Base on this information, the first action the nurse should take is to:

a. Deflate the esophageal balloonb. Encourage him to take the deep breath c. Monitor his vital signs d. Notify the physician

14. Because the detoxification of alcohol damages tissues a high-calorie diet,

Page 33: Document1

fortified with vitamins should be encouraged to protect Mr. Silverio's:

a. Liver b. Kidneysc. Adrenals d. Pancreas

Situation 4 - Rape is one of the most tragic things that could happen to anyone especially with young girls. Incidence such as these could develop into a crisis situation involving not only the rape victims but also their families.

15. This type of crisis could be an example of which of the following?

a. Combination of developmental and situational b. Situational c. Emotional d. Developmental

16. Noemi, a staff nurse in the emergency room, realizes that she has an important role to play as a patient advocate to rape victims. To demonstrate this role, she takes note of one of the responsibilities?

a. Since this is a legal case, call the press about the incidence of rape b. Perform thorough physical assessment and documenting objectively all the evidences of rape c. Ask the patient to stay in one isolated room first to provide privacy while attending to other patientsd. Provide emotional support first and postponed physical assessment when patient is already calm

17. Which of the following is a form of active, focused, emotional environmental first aid for patients in crisis?

a. Attitude therapy b. Psychotherapy c. Crisis intervention d. Re-motivation technique

18. Which of the following is true with regards to crisis?

a. Crisis is self-limitingb. After crisis, the individual always return to a pre-crisis state or condition c. Crisis always result in adaptive behavior d. The person in crisis is not susceptible for

any help

19. If help is not provided in a crisis situation, an individual may spontaneously resolve in negatively or positively by returning to pre-crisis state, usually within which of the following duration?

a. 2-3 weeks b. 3-4 weeks c. 1-2 weeks d. 4-6 weeks

Situation 6 - One Important fact that will guide the nurse in the practice of the profession is her knowledge of the nursing law.

20. The nurse practice Act of 1991 regulates the practice of nursing in the Philippines. Which of the following statements about this Act is true?

a. This Act delineates the practice of nursing and midwiferyb. It was enacted in November 1991c. The primary purpose is to protect the publicd. The Act defines the practice of nursing in the Philippines

21. When a nurse starts working In a hospital but without a written contract, which of the following is expected of her?

a. She's not bound to perform according to the standards of nursing practice b. Provides nursing care within the acceptable standards of nursing practice c. She's not obligated to provide professional service d. The employer does not hold the nurse responsible for her action

22. A patient, G8P5, refused to be injected with the 3rd dose of Depoprovera. Thenurse insisted inspite of the patients refusal and forcibly injected the contraceptive. She can be sued for which of the following?

a. Misrepresentationb. Assault and Battery c. Malpractice d. Negligence

23.A patient has been in the ICU for 2 weeks. The relatives have consented to a "Do not resuscitate order," When the patient develops a cardiac arrest, the nurse will carry out which of the following actions?

a. Only medication will be givenb. All ordinary measure will be stoppedc. Basic and advance life support will not be given d. Mechanical ventilation and NGT will be stopped

24. When a patient falls from bed, which of the following is your immediate action?

a. Report to the head nurse and calls someone to help b. Determine any injury or harm c. Refer to the resident on duty d. Put back patient to bed

Situation 7 - Ms. May Mansur encountered vehicular accident on her way to the office and he remains conscious. Police officers brought her to the hospital.

25. You have to observe for increase intracranial pressure. Which of the following is not a sign of increased intracranial pressure?

a. Headacheb. Vomiting c. Vertigo d. Changes on the level of consciousness

26. Which of the following drug may be given to reduce increase intracranial pressure?

a. Scopalamineb. Lanoxin c. Coumadin d. Mannitol

27. Since she medicated to reduce increased intracranial pressure. What nursing measure must be done to prevent further complication?

a. Encourage her to observe bed restb. Check blood pressure every shift c. Observe complete best rest d. Measure intake and output

28. In what manner would you be able to

Page 34: Document1

assess accurately her motor strength?

a. Observe how he talksb. Instruct her to squeeze her hands c. Allowing him to stand alone d. Pricking her skin with pin

29.Which of the following activities would cause her a risk in the increase of intracranial pressure?

a. Coughing b. Readingc. Turning d. Sleeping

Situation 8 - Basic Psychiatric concepts a nurse should be aware of...

30. Mental experiences, operate on different levels of awareness. The level that best portrays one's attitudes, feelings, and desire is the:

a. Conscious b. Unconscious c. Preconscious d. Foreconscious

31. The ability to tolerate frustration is an example of one of the functions of the:

a. Id b. Ego c. Superego d. Unconscious

32. In the process of development the individual strives to maintain, protect, and enhance the integrity of the self. This normally accomplished through the use of:

a. Affective reactionsb. Ritualistic behaviorc. Withdrawal patterns d. Defense mechanisms

33. Sublimation is a defense mechanism that helps the individual:

a. Act out in a reverse something already one or thoughtb. Return to an earlier, less mature stage of developmentc. Exclude fro the conscious things that are psychologically disturbingd. Channel an acceptable sexual desire into socially approved behavior

34. An example of displacement is:

a. Imaginative activity to escape realityb. Ignoring unpleasant aspects of realityc. Resisting any demands made by othersd. Pent-up emotion directed to other than the primary source

Situation 9 - Joan, age 34, is hospitalized because of alcoholism.

35. Joan denied that she has a problem with alcohol. The nurse understands that Joan uses denial for which of the following reasons:

a. To reduce her feelings of guilt b. To iive up to others' expectationc. To make her seem more independentd. To make her look better in the eyes of others

36. Joan appears suspicious of others and blames them for her personal problems. The nurse understands the client is using this behavior because which of the following difficulties?

a. In telling the truthb. Meeting an ego ideal c. With dependence and independenced. In identifying who is creating the problem

37. When thinking about alcohol and drug abuse, the nurse is aware that:

a. Most polydrug abusers also abuse alcohol b. Most alcoholics become polydrug abusersc. Addictive individuals tend to use hostile abusive behaviord. An unhappy childhood is a causative factor in many addictions

38. The most important factor in rehabilitation of a client addicted to alcohol is:

a. The availability of community resources b. The accepting attitude of the client’s familyc. The client's emotional or motivational readiness d. The qualitative level of the client's physical state

39. Joan asks if attendance of Alcoholics Anonymous is required. Which of the following would reflect the nurse's reply?

a. "You'll find you need their support."b. "Do you have feelings about going to these meetings?" c. "No its best to wait until you feet you really need them."d. "Yes, because you will learn how to cope with your problem."

Situation 10 - Nurse Medie has been encountering schizophrenic and different psychotic disorders. .

40. A male client who has delusions of persecution and auditory hallucination is admitted for psychiatric evaluation after stabbing a friend. Later a nurse on the unit greets the client by saying, "Good evening. How are you?" The client who has been referring to himself as "man," answers, "The man is bad." This is example of:

a. Dissociation b. Transference c. Displacement d. Reaction formation

41. A disturb client starts to repeat phrase that others have just said. This type of speech is known as:

a. Autismb. Echolalia c. Neologism d. Echopraxia

42. Projection, rationalization, denial, and distortion by hallucinations and delusions are examples of a disturbance in:

a. Logic b. Association c. Reality testing d. The thought process

43. The major reasons for treating severe emotional disorders with tranquilizers is to:

a. Reduce the neurotic syndrome b. Prevent secondary complication c. Prevent destructiveness by the client d. Make the client more amenable to psychotherapy

Page 35: Document1

44. The nurse recognizes that dementia of the Alzheimer's type is characterized by :

a. Aggressive acting out behaviorb. Periodic remissions and exacerbations c. Hypoxia of selected areas of brain tissue d. Areas of brain destruction called senile plaques

Situation 11 - Aisa, is a 4-year old with severe anemia. She is seen by the nurse in the clinic.

45. In addition to weakness and fatigue, which of the following problems should the nurse expect Aisa to exhibit?

a. Cold, clammy skin b. Increased pulse rate c. Elevated blood pressure d. Cyanosis of the nail beds

46. Which of the following problems associated with anemia best explains why Aisa becomes dizzy during periods of physical activity?

a. An inflammation of the inner earb. Insufficient cerebral oxygenation c. A sudden drop in blood pressured. Decreased level of serum glucose

47. Aisa is to receive a liquid iron preparation. Which of the following directions would be appropriate for the nurse to teach Aisa's mother?

a. Administer this at least an hour before meals b. Explain that loose stools are common with ironc. Have Aisa take the diluted iron preparation through a straw d. Avoid giving Aisa orange or other citric juices with the iron preparation

48. Aisa is to have blood transfusion. Which of the following problems is most likely associated with blood transfusion?

a. Serum hepatitisb. Allergic response c. Pulmonary edema d. Hemolytic reaction

Situation 12 - Eric Pineda is admitted to hospital to have his urethra dilated by the physician. A urinary retention catheter is inserted following the procedure.

49. A routine urinalysis is ordered for Mr. Pineda. If the specimen cannot be sent immediately to the laboratory, the nurse should:

a. Take no special action b. Refrigerate the specimen c. Store on dry side of utility roomd. Discard and collect a new specimen later

50. The nurse understands that the structure that encircles the male urethra is the:

a. Epididymis b. Prostate gland c. Seminal vesicled. Bulbourethral gland

51. The nurse can best prevent the contamination from Mr. Pineda's retention catheter by:

a. Perineal cleansing b. Encouraging fluids c. Irrigating the catheterd. Cleansing around the meatus periodically

52. When Mr. Pineda, who has urinary retention catheter in place, complaints of discomfort in the bladder and urethra the nurse should first:

a. Notify the physicianb. Milk the tubing gentlyc. Check the patency of the catheter d. Irrigate the catheter with prescribed solutions

53. Mr. Pineda experiences difficulty in voiding after his indwelling urinary catheter is removed. This is probably related to:

a. Fluid imbalanceb. Mr. Pineda's recent sedentary lifestylec. An interruption in normal voiding habits d. Nervous tension following the procedure

Situation 13 - Helen Alcantara is admitted to hospital with complaints of hematuria, frequency, urgency, and dysuria.

54. Mrs. Alcantara's signs and symptoms would most likely be associated with:

a. Pyelitis b. Cystitis c. Nephrosis d. Pyelonephritis

55. Mrs. Alcantara has a higher risk of developing cystitis than does a male. This isdue to:

a. Altered urinary pH b. Hormonal secretions c. Position of the bladder d. Proximity of the urethra and anus

56. The family of an elderly, aphasic client complain that the nurse failed to obtain a signed consent before insertion of indwelling catheter to measure hourly output. This is an example of:

a. A catheter inserted for the client's benefitb. A treatment that does not need a separate consent form c. Treatment without consent of the client, which is an invasion of rightsd. Inability to obtain consent for treatment because the client was aphasic

57. When caring for a client with continuous bladder irrigation, the nurse should:

a. Monitor urinary specific gravityb. Record urinary output every hourc. Subtract irrigant from output to determine urine volume d. Include irrigating solution in any 24 hour urine tests order

58. When urinary catheter is removed, the client is unable to empty the bladder. A drug is used to relieve urine retention is:

a. Carbachol injection b. Neosporin GU irrigant c. Bethanecol (Urecholine) d. Pilocarpine hydrochloride (Pilocar)

Situation 14 - Arman Adriatico is admitted to hospital with extensive carcinoma of the descending portion of the colon with metastasis to the lymph nodes.

59. The operative procedure that would probably be perform to Mr. Adriatico is a

Page 36: Document1

(an):

a. lleostomyb. Colectomy c. Colostomyd. Cecostomy

60. The primary step toward long-range goals in Mr. Adriatico's rehabilitation involves his:

a. Mastery of techniques of ostomy careb. Readiness to accept an altered body function c. Awareness of available community resourcesd. Knowledge of the necessary dietary modifications

61. When teaching Mr. Adriatico to care for a new stoma, the nurse should advice him that irrigations be done at the same time every day. The time selected should:

a. Be appropriate hour before breakfastb. Provide ample uninterrupted bathroom use at home c. Approximate Mr. Adratico's usual daily time for eliminationd. Be about halfway between the two largest meals of the day

62. When performing the colostomy irrigation, the nurse inserts the catheter into the stoma:

a. 5cmb.10cm c.15cm d.20cm

63.Mr. Adriatico should follow a diet that is:

a. Rich in protein b. Low in fiber content c. High in carbohydrate d. As close to normal possible

Situation 15 - Richard Gabatan, a 32-year-old car salesman, suffered a spinal cord injury in a motor vehicle accident resulting to paraplegia.

64. A nurse finds Mr. Gabatan under the wreckage of the car. He is conscious, breathing satisfactorily, and lying on the back complaining of pain in the back and an inability to move his legs. The nurse

should first:

a. Leave Mr. Gabatan lying on his back with instructions to move and then go seek additional helpb. Gently raise Mr. Gabatan to a sitting position to see if the pain eitherc. Roll Mr. Gabatan on his abdomen, place, a pad under his head, and coverhim with any material available d. Gently lift Mr. Gavatan into a flat piece of lumber and using any available transportation, rush him to the nearest medical institution

65. Once admitted to hospital the physician indicates that Mr. Gubatan is a paraplegic. The family asks the nurse what that means. The nurse explains that:

a. Upper extremities are paralyzedb. Lower extremities are paralyzed c. One side of the body is paralyzed d. Both lower and upper extremities are paralyzed

66. The nurse recognizes that the major early problem for Mr. Gabatan will be:

a. Bladder control b. Client education c. Quadriceps setting d. Use of aids for ambulation

67. The nurse should expect Mr. Gabatan to have some spasticity of the lower extremities. To prevent the development of contractures, careful consideration must be given to:

a. Active exercise b. Deep massage c. Use of tilt board d. Proper positioning

68. Rehabilitation plans for Mr. Gabatan;

a. Should be left up to Mr. Gabatan and his familyb. Should be considered and planned for early in his care c. Are not necessary, because he will return to former activitiesd. Are not necessary, because he will probably not able to work again

Situation 16- Karen Boltron, age 16, is withdrawn and non communicative. She

spendsmost of her time lying on her bed.

69. Which nursing intervention would be the most appropriate way to help Karen accept the realities of daily living?

a. Assist her to care for personal hygiene needs b. Encourage her to keep up with school studies c. Encourage her to join the other clients in group singing d. Leave her alone when these appears to be a disinterest in the activities at hand

70. Which is the best plan of nursing intervention to encourage Karen to talk:

a. Try to get her discuss feelingsb. Focus oh non threatening subjects c. Ask simple questions that require answersd. Sit and look magazines with her

71. Which of the following is an important aspect of nursing intervention when caring for Karen?

a. Help keep her oriented to realityb. Involve her in activities throughout the day c. Encourage her to discuss why mixing with other people is avoidedd. Help her understand that it is harmful to withdraw from situations

72. One day Karen suddenly walks up to the nurse and shouts. "You think you're so damned perfect ad good. i think you stink," Which response should the nurse make?

a. "You seem angry with me." b. "Stink? I don't understand." c. "Boy, you're in a bad mood." d. "I can't be all that bad, can I?"

73. On being discharged, a client with psychiatric problems should be encouraged to:

a. Go back to regular activities b. Call the unit whenever upsetc. Continue in an after care situationd. Find a group that has similar problem

Situation 17 - Danny Dasigao, age 63, has an obsessive-compulsive behavior

Page 37: Document1

disorder. He believes that the doorknobs are contaminated and refuses to touch them except with the tissue.

74. Which intervention should the nurse make when dealing with Danny's fear of doorknobs?

a. Supply rim with paper tissue to help him function until his anxiety is reduced b. Explain to him that this idea about doorknob is part of his illness and is not necessaryc. Encourage him to scrub the doorknobs with a strong antiseptic so he does not need to use tissues d. Encourage him to touch doorknobs by removing all available paper tissue until he learns to deal with the situation

75. Which stimulus is possibly motivating Danny to use paper towels to open doors?

a. He is using the method to punish himself

b. He is listening to voices telling him that the doorknobs are unclean c. He wants to unconsciously control unacceptable impulses or feelings d. He has a need to punish others by carrying out an annoying procedure

76. Which action by the nurse would most likely decrease Danny's anxiety?

a. Explore with him the nature of his anxiety b. Stimulate him to express his ritualistic actions regularly c. Encourage him to participate in his therapeutic plan of care d. Provide him with an environment that is both supportive and non-opinionated

77. Which intervention should be included in Danny's initial treatment plan?

a. Deny his time for the ritualistic behavior b. Give a schedule for the ritualistic behavior c. Determine the purpose of the ritualistic behavior d. Suggest a symptom substitution technique to refocus the behavior

78. The most appropriate way to decrease a clients anxiety is by:

a. Avoiding unpleasant objects and eventsb. Prolonged exposure to fearful situationc. Acquiring skills with which to face stressful events d. Introducing an element of pleasure into fearful situations

Situation 18 - Jennifer Yadao, age 16, is admitted with the diagnosis of anorexia nervosa. She has lost 10 kg in 5 weeks. She is very thin but excessively concerned about being overweight. Her daily intake is 10 cups of coffee.

79. Which nursing intervention should the nurse initially perform for Jennifer? a. Explain the value of good nutritionb. Compliment her on her lovely figure c. Try to establish a relationship of trust d. Explore the reasons why she does not eat

80. Which stimulus is the most likely cause of Jennifer's disorder?

a. Allow self-esteem b. Feelings of unworthinessc. Anger directed at the parentsd. An unconscious fear of growing up

81 Jenifer is to be placed on behavior modification. Which is appropriate to include in the nursing care plan?

a. Remind frequently the client to eat all the food served on the tray b. Increase phone calls allowed the client by or a per day for each pound gained c. Include the family with the client in therapy sessions two times per week d. Weigh the client each day at 6:00 A.M. in hospital gown and slippers after she voids

82. Another patient, Kara, 17 years old, is also diagnosed with anorexia nervosa. You have been assigned to sit with her while she eats her dinner. Kara says to you, "My primary nurse trusts me. I don't see why you don't." Your best response is:

a. "I do trust you, but S was assigned to be with you."b. "It sounds as if you are manipulating me." c. "OK. When S return, you should have eaten everything."

d. "Who is your primary nurse."

83. Which observation of the client with anorexia indicates that the client is improving?

a. The client eats meals in the dining roomb. The client gains one pound per week c. The client attends group therapy sessionsd. The client has a more realistic self-control

Situation 19 - Mr. Pascua is pacing about the unit and wringing his hands. He is breathing rapidly and complains of palpitations and nausea and he has difficulty focusing on what the nurse is saying. •

84. Mr. Pascua is experiencing a high degree of anxiety. It is important to recognize if additional help is required because:

a. If the client is out of control, another person will help to decrease his anxiety level b. Being alone with an anxious client is dangerous c. It will take another person to direct the client into activities to relieve anxietyd. Hospital protocol for handling anxious clients requires at least two people

85. He says he is having a heart attack but refuses to rest. The nurse would be Interpret his level of anxiety as:

a. Mild b. Moderatec. Severed. Panic

86.What should the nurse include in the care plan to Mr. Pascua when he is havinga panic attack?

a. Calm reassurance, deep breathing and modication as ordered b. Teach Mr. Pascua problem solving in relation to his anxiety c. Expiam the physiologic responses of anxietyd. Explore alternate methods for dealing with the cause of his anxiety

Situation 20 - Joel is a toddler who has classical hemophilia.

Page 38: Document1

87. Which of the following statements is true regarding Joel's disorder?

a. Hemophilia is an autosomal dominant disorder in which the woman carriesthe traitb. Hemophilia follows regular laws of Mendelian inherited disorders such as sickle ceil anemiac. This disorder can be carried by either male or female but occurs in the sex opposite that of the carrier d. Hemophilia is an X-linked disorder in which the mother is usually the carrier of the illness but is not affected by it

88. Joel has some internal bleeding. At which of the following sites is the most common for the child with hemophilia to bleed?

a. Joints b. Intestines c. Cerebrum d. Ends of the log bones

89. Which of the following blood products is most likely to be given to Joel?

a. Albuminb. Fresh frozen plasma c. Factor VIII concentrate d. Factor II, Vll, IX, X complex

90. Joel's parents ask if-their other children will be affected by the disorder. Which of the following statements should guide the nurse in her response?

a. All the girls will be normal and the other son a carrier b. All the girls will be carriers and one half the boys will be affected c. Each son has a chance of being affected and each daughter a 50% chance of being a carrierd. Each son has 50% chance of being affected or a carrier, and the girls will be all carriers.

91. A child is to receive a blood transfusion, if an allergic reaction to the blood occurs, the nurse's first intervention should be:

a. Call the physicianb. Slow the flow ratec. Stop the blood immediately

d. Relieved the symptoms with an ordered antihistamines

Situation 19 - Mr. Villa who was admitted to the respiratory floor with COPD. The nurse finds him extremely restless, incoherent, and showing signs of acute respiratory distress. He Is using accessory muscles for breathing and Is diaphoretic and cyanotic.

92. The best initial action by the nurse is to:

a. Administered oxygen as ordered b. Assess vital signs and neural vital signs c. Administered medication which has been ordered for paind. Call respiratory therapy for a prescribed ABG (arterial -blood gas) analysis

93. An order is written for oxygen by nasal cannula at 2 liters per minute. Which assessment is most useful in assessing the adequacy of the oxygen therapy?

a. Respiratory rateb. Color of mucus membranesc. Pulmonary function testsd. Arterial blood gases

94. Mr. Villa needs frequent monitoring of arterial blood gases. Following the drawing of arterial blood gasses it is essential for the nurse to do which of the following?

a. Encourage the client to cough an deep breath b. Apply pressure to the puncture site for 5 minutes c. Shake the vial of blood before transporting it to the lab d. Keep the client on bed rest for 2 hours

95. The nurse is interpreting the results of a blood gas analysis performed on an adult client. The value include pH of 7.35, pC02 of 60, HC03 of 35. and 02 of 60. Which interpretation is most accurate?

a. The client is in metabolic acidosisb. The client is in compensated metabolic alkalosisc. The client is in respiratory alkalosisd. The client is in compensated respiratory acidosis

Situation 20 - The nurse is assigned in a counseling clinic about preventive measures for cancers.

96. Cancer is the second major cause of death in this country. What is the first step toward effective cancer control?

a. Increasing governmental control of potential carcinogensb. Changing habits and customs that predispose the individual to cancerc. Conducting more mass screening programsd. Educating public and professional people about cancer

97. In order to educate clients, the nurse should understand that the most common site of cancer for a female is the:

a. Uterine cervix b. Uterine body c. Vagina d. Fallopian tube

98.A client has just completed a course in radiation therapy and is experiencing radio-dermatitis. The most effective method of treating the skin is to:

a. Wash the area with soap and warm water b. Apply a cream or lotion to the areac. Leave the skin alone until it is clear d. Avoid applying creams or lotion to the area

99.A client with cancer that has metastazised to the liver is started on chemotherapy- His physician has specified divided doses of the antimetabolite. The client asks why he could take the drug in divided doses. The appropriate response is:

a. " There really is no reason your doctor just wrote the orders that way." b. "This schedule will reduce the side effect of the drug." c. "Divided doses produce greater cytotoxic effects on the diseased cells." d. "Because these drugs prevent cell division, they are more effective in divided doses,"

100. A client has possible malignancy of the colon, and surgery is scheduled. The

Page 39: Document1

rationale for administering Neomycin preoperatively is to:

a. Prevent infection postoperativelyb. Eliminate the need for preoperative enemasc. Decreased and retard the growth of normal bacteria in the intestinesd. Treat cancer of the colon

Page 40: Document1

1. When can trained hilot attend to a delivery?

a. When at a time of delivery, no licensed personnel trained on maternal care is around b. When the patient is living in a remote areac. When the worker is considered to have a high risk pregnancyd. The mother requested the hilot to attend to the delivery

2. What mineral supplement is given to a pregnant woman in the third trimester of pregnancy?

a. Vit.Cb. Fe c. Cad. FolicAcid

3. The following are benefits of breastfeeding to the infant except:

a. Provides a nutritionally complete food for the young infantb. Strengthens the infants immune system, preventing many infections c. Reduces the infant's exposure to infection d. Provides a natural method of delaying pregnancies

4. Perception of a toddler about illness is:

a. Life threatening b. Punishment for wrong doings c. A necessary part of lifed. The will of god

5. Most common manifestation of Anemia

a. Weight lossb. Fatigue c. Ahorexiad. Poor digestion

6. The following are needs of infants

I. ComfortII. SleepIII. Stimulation IV. Modern paraphernalia

a. I, III, IVb. I, II, IV

c. II, III, IV d. I, II, III

7. Which drug is not contraindicated to a patient in a first trimester of pregnancy.

a. Paracetamolb. Cytoxan c. Terbutahed. Calcium Lactate

8. Which can be given to a patient with hyperemesis gravidarum to relieve the signs & symptoms?

a. Have the patient eat crackers before rising from the bed in the morning b. Offer water with mealsc. Let the patient choose what she feels like eating d. Let the patient stay on bed until the feelings of nausea subsides.

9. When can you inform an adopted child that he/she is adopted?

a. When he/she asks where he/she came from. b. Wait until the child reaches age of majority. c. When he/she shows interest in story telling.d. Watery and greenish

10. A new mother asks the nurse to describe the normal stool pattern of a breast-fed neonate. The nurse correctly describe the stool as being:

a. Watery and golden yellowb. Pasty and yellow c. Thick, black, and odoriesd. Watery and greenish brown

11. For immunization of pregnant women, which tetanus toxoid is given as early as possible during pregnancy?

a. TT3 b. TT4 c. TT1 d. TT2

12. A woman uses a diaphragm as contraceptive. You would instruct her to return to the clinic to have hem diaphragm fit checked after which of the following circumstances?

a. Cervical infection b. A weight gain of 20 pounds c. A vaginal infectiond. Six n-onths of non-use

13. When a woman uses a vaginal spermicide, which of the following techniques should she use?

a. Coitus should be followed by a douche within 6 hours b. Insert the product by applicator no more than 1 hour prior to coitus c. Keep a feminine hygiene product available to use in case her supply of spermicide runs outd. Place the product near the vaginal orifice for immediate contact instead of back of the vagina.

14. The hormone that is secreted by the corpus luteum and prepares the endometrium implantation is:

a. Estrogen b. Luteinizingc. Progestorene d. Prostaglandin

15. Which hormone causes spinnbarkeit and ferning to occur?

a. FS H b. Gn RHc. Progesterone d. Estrogen

16.The pregnant patient is in her third month when she makes her second prenatal visit. She asks the nurse "what is happening right now in the development of the baby?" The nurse answer:

a. "The heart is beginning to pump bloods." b. "The brain is dividing into section,"c. "Lanugo and vernix caseosa are forming to protect the embryo."d. "The embryo is becoming a fetus and sex is determined."

17.A new patient states, "I must be about four months pregnant. I cannot remember exactly my last menstrual period, but I have been feeling the baby kicking for 3 to 4 weeks now." Upon auscultation the nurse hears fetal heart sounds. The nurse stales that this assessment indicates:

Page 41: Document1

a. "You are not quite four months." b. "You are in your fifth months." c. "You may be six months along." d. "You are probably seven months pregnant."

18. The nurse is doing an initial assessment with the pregnant patient who states that she is a strict vegetarian. What vitamin supplement should be recommended?

a. Ab. C c. Dd. B12

19.The best method to prevent hemorrhage after caesarian birth is to:

a. Provide regular analgesics to enhance urination b. Reposition the woman from side to side c. Observe vital signs for falling blood pressure d. Assess the uterine fundus regularly for firmness

20.When teaching the postpartum woman about peripads, the nurse should tell her that:

a. She can change tampons when the initial perineal soreness goes away b. Pads having cold packs within them, usually hold more lochia than regular pads c. Blood-soaked pads must be returned in a plastic bag to the hospital after discharged. The pads should be' applied and removed in a front-to-back direction

21 .To prevent breast engorgement, the nurse should teach the non-lactating postpartum woman to:

a. Maintain loose-fitting clothing over her breast b. Pump the breast briefly if they become painful c. Limit fluid intake to suppress milk production d. Wear a well-fitting bra or breast binder constantly

22. The woman who has just completed her 13th weeks of pregnancy comes in for her monthly visit. The nurse expresses concern regarding the weight gain. The

patient asks, 'Well, how much weight should I have gained by now? The answer is:

a. "About 10lbs." b. “Not more than 2 to 3 Ibs.” c. "Not more than 10 Ibs." d. "No weight gain is expected this soon."

23. The pregnant patient asks the nurse when she should start breastfeeding. The nurse replies:

a. "When your milk comes down."b. "As soon as possible after delivery." 'c. "In two or three days when you are feeling better."d. "I do not recommend breast-feeding."

24.The patient is being administered oxytocin at 14 mi U/min. At what point would the nurse DC the infusion.

a. If the patient is not going into labor b. If the patient is in too much painc. Contractions lasting more than 40 seconds d. Contractions lasting more than 6 seconds

25. In explaining the pattern of discharge following delivery, the nurse explains than lochia will be heavier.

a. in the morning b. at night c. as lochia cessation nears d. toward the end of lactation

26. Choose correct pre-operative teaching before planned caesarian birth.

a. Oral intake will be limited to clear fluids for 12 hours before surgery b. Intravenous fluids are usually continued for two days after birthc. The woman will be asked to take deep breaths and cough regularly after birth. d. The nurse will help her ambulate to the restroom to urine within 4 hours of birth.

27. Twelve hours after birth, a mother a lies in bed resting although she will be discharged in another 12 hours, she does not ask about her baby provide any care. What is the probable reason for her behavior?

a. She is still in the taking in phase

maternal adaptation b. She shows behaviors that may lead to postpartum depression c. She is still affected by medications given during labord. She may be dissatisfied with some aspect of the new born

28.A newborn is rooming in with his teenage mother, who is watching TV. The nurse notes that the baby is awake and quite.

a. Pick the baby up and point out his behaviors to the mother b. Tell the mother to pick up her baby and talk with him while he is awakec. Focus care on the mother, rather than the infant so that she can recuperated. Encourage the mother to feed the infant before he begin crying

29. A woman is considering having a tubal ligation after she giyes birth to her second child. The nurse should counsel her that:

a. She should breast feed until several months after birth to be certain that the infant is healthyb. The procedure should be considered permanent and irreversiblec. Steralization is an easier procedure to perform after the postpartum period

30. How should a woman take oral contraceptives?

a. On an empty stomach with a full glass of waterb. At about the same time each dayc. Before every episode of intercourse d. In the morning and at bed time

31. Choose the safety teaching related oral contraceptives?

a. A barrier method should also be used to protect from infectionb. Nausea suggests that stroke may be imminentc. A toxic shock syndrome is more likely to occur when the pill is usedd. Increase fluids if urinary frequency or urgency occurs

32. The intrauterine device is an appropriate contraceptive for the woman who:

Page 42: Document1

a. Has unplanned intercourse with several partners b. Was recently hospitalized for treatment of a pelvic infection c. Is in mutually monogamous relationship d. Has had two ectopic pregnancies

33. The woman who is receiving methorexate for an ectopic pregnancy should be cautioned to avoid:

a. Driving or operating machinery b. Eating raw vegetables or fruits c. Using latex condoms for intercourse d. Taking vitamin with folic acid

34.To reduce the risk for toxic shock syndrome, women should be taught to:

a. Avoid changing tampons until they are thoroughly saturatedb. Use a diaphragm with spermicidal jelly during the menstrual periodc. Wash hand thoroughly before inserting a tampon or diaphragm into the vagina d. Limit the use of super absorbent tampons to the times when the flow is heavy

35. Cervical mucus at ovulation should be:

a. Thin, slippery, and should stretch to at least 6 centimeter b. Cloudy, with a mild odor, and should stretch to at least 6 centimeter c. Thick, clear, and of a large quantity d. Thin and tinged with a small of blood

36. An ovulation, the basal body temperature usually:

a. Rises abruptly and then falls after 1 or 2 days b. Falls and remains low for the last half of the cycle.c. Is higher during the first half of the cycle than in the last half d. Falls slightly at ovulation and is higher during the last half of the cycle

37. The goals of maternal and child health nursing are:

I. That every child lives and grows up in a family unit with love and securityII. To ensure that every that every expected and nursing mother maintain good health.

III. To ensure that every mother has a normal delivery and bears healthy children. IV. To achieve healthy sexual development and maturation.

a. I, II, III b. I, III, IV c. I, II, IV d. II, III, IV

38. Based on the DOH program, a mother should have at least how many prenatal visits during preqnancy?

a. 3 b. 4c. 5d. 12

39. A standard prenatal physical examination per visit should be performed. Which of the following is not included in the routine examination?

a. BP . b. Height c. Lymph nodes d. conjunctive of the eyes

40. In goiter endemic areas, all pregnant women shall be given one iodized oil capsule every: . a. 6 monthsb. 1 year c. 3 months d. 4 months

41. In the care of "high risk" pregnant women, "tagging" the prenatal record means:

a. Placing a name tag around patients wrist.b. Assigning the mother under "high risk" group c. Writing the letters "HR" in red ink against the entry in the prenatal register d. Writing the name of the patient in red ink in the prenatal register

42. In areas where licensed heath personnel are not available, who shall be trained to regular prenatal visits using the Home Based mother's Record to identify danger signs:

a. Midwife

b. Trained hilots c. Rural Health Nurse d. Barangay officials

43. The following are qualified for home.delivery:

I. full term II. more than 4 pregnancies III. Adequate pelvis IV. Cephalic presentation

a. I, II, IIIb. I, III, IVc. II, III, IVd. I, II IV

44. A home delivery hit should contain a complete set of gadgets needed during delivery. Which of the following is optional?

a. suction bulk b. boiled razor blade c. pair of scissors d. clean towel

45. This provide a valuable index for evaluation of the newborn infant’s condition at birth:

a. APGAR score b. Muscle tone c. Respiratory effort d. Heart rate

46.The major cause of maternal deaths is:

a. prolonged labors b. exhaustionc. hemorrhaged. infection

47. When giving a nursing care to a mother after delivery, the following should be checked, except:

a. uterus is contracted and hard b. BP, and pulse rate is normalc. Placenta must be complately expelledd. Milk production is adequate

48. Which of the following are risk factors for pregnancy?

I. age-under 18 y.o.ll. height-less than 145 cm tall lII. 4th pregnancy IV. History of previous caesarian section

Page 43: Document1

a. I, II, IIIb. I, III, IV c. I, II, IVd. II, III, IV

49. Any abnormally detected during physical examination of the newborn should be reported to the physician. This should include:

a. head circumference 34 cmb. weight- 2,350 cmsc. chest circumference- 33 cm d. length – 49.5 cm

50. This law requires compulsory immunization against hepatitis B for infants and children below eight (8) years old.

a. P.D. 996b. RA 7846c. Presidential Proclamation No. 1066d. P.D. 651

51. Which of the following immunization can be given any time after birth?

a. BCGb. DPTc. OPVd. Measles

52. When assessing a 3 month old infant . Which of the following will you expect to find?

I. Smiles spontaneously II. Rotate the head from side to sideIII. Sits without support IV. briefly holds toy in hand

a. I, II, IIIb. I, III, IVc. II, III, IVd. I, II, IV

53.The following are strategies used for the attainment of goals of the DOH Dental health program, except:

a. Social mobilizationb. Networking with other servicesc. Monitoring feedbackd. Home visits

54. This project will be a continuous solicitation of donation for new kiddie toothbrushes:

a. 2 year care program,b. Orientation Training on Comprehensive dental health program c. "Sang Milyong Sipilyo Project" d. Dental Health Services Clinic

55. A mother together with her 3 year old daughter came to the Dental Health clinic for check up. Which of the following is not one of the direct services offered by the Dental Health Program.

a. Dental curative Program b. Oral Habilitation and Rehabilitation Program c. Training Program d. Dental Preventive Program

56. When assessing a.neonate a few hours after birth, the nurse notes an edematous area over the pariental are that does not cross the sagittal suture line, This is most likely indiates

a. caput succedaneumb. cranlosyntostosisc. cranlotablesd. cephalhematoma

57. Mrs. P. has finished feeding her 5-day old neonate but is having difficulty burping him. Which instructions should the nnurse give her?

a. "Give him water and hold him on his side"b. "Hold him upright against your shoulder and pat his back"c. Give his pacifier and hold im face down"d. Hold him with his head slightly elevated and rub his stomach"

58. Which is the most reliable early indicator of neonated infection?

a. An elevated temperatureb. A change in feeding patternc. A palpable massd. Excessive mucos

59. When bathing a newborn, the nurse should be especially careful to:

a. Avoid cleaning his umbilical cord stumpb. Immerse him in warm water onlyc. Wash his scalp every dayd. Keep him warm

60 A mother asks the nurse about scheduling her daughter for routine immunizations. At which age should her baby receive her initial dose of the diptheria - pertussis - tetanus vaccine?

a. 1 monthb. 2 monthsc. 4 monthsd. 6 months

61. A decrease in the baseline FHR may be caused by all of the following factors except:

a. Fetal sleepb. Fetal hypoxiac. Maternal drug administrationd. Maternal fever

62. Which assessment findng is not a contraindication for using tocolytic agent to manage preterm labor?

a. Active vaginal bleedingb. fetal distressc. Cervical dilatation of 2 to 3 cmd. Cervical dilatation of 4 to 5 cm

63. Which assesement finding indicates hypoglycemia in a neonate?

a. Tremorsb. Projectile vomitingc. Diarrhead. jaundice

64. Which adverse effect may occur in a patient receiving bromocriptine mesylate (parlodel) to prevent postpartal lactation?

a. Hypotensionb. Tachycardiac. Bradycardiad. Breast engorgement

65. Mrs Bugna, a 25 year old has missed 2 menstrual periods and is making her initial visit to the antepartal clicnic. Her last mentrual period began on June 3. Using the nagele's rule, the nurse would calculate her expected date of delivery as:

a. April 3b. March 24c. March 10d. February 24

(Questions to 66 to 72 refer to this

Page 44: Document1

situaltion)

Situation: Mrs Og gravida 2 para 1 is accompanied to the labor and delivery are by her husband. Both have attended lamaze classes. Initial assessment reveals cervical dilation of 5 cm. Cervical assessment 80% station, -3; duration of contractions, 40 to 50 seconds; frequency of contractions 5 to 8 minutes; membranes ruptured spontaneousely 1 hour before admission; presentation vertex and possition left occiput anterior (LOA). Mrs Og, is connected to an external fetal monitor;

66. Based on the initial assessment findings, the fetal presenting part is:

a. At the level of the pelvic inletb. At the level of the ischial spinesc. 1 cm below the iscial spinesd. At the perineum

67. The fetal heart rate should be most audible in which dominant quadrant?

a. Left upper quadrantb. Left lower quadrantc. Right upper quadrantd. Right lower quadrant

68.The LOA position means that the:

a. Lie is longitudinal and the fetal occiput is directed toward the left posterior portion of the maternal pelvisb. Lie is transverse and the fetal mentum is directed toward the left posterior portion of the maternal pelvis c. Lie is longitudinal and the fetal occiput is directed toward the left anterior portion of the maternal pelvis. d. Lie is oblique and the fetal anterior fontanel is directed toward the left posterior portion of the maternal pelvis

69. Which assessment finding would necessitate bedrest for Mrs. Og?

a. 5cm cervical dilation b. 80% cervical effacement c. Contractions every 5 to 8 minutes d. 3 station

70. The fetal monitor strip shows an FHR deceleration occurring during the increment of the contraction, reaching its lowest point at the acme of the

contraction, and returning to the baseline during the decrement of the contraction. This type of deceleration:

a. indicates fetal distressb. Is caused by uteroplacental insufficiencyc. Indicates fetal vagal nerve stimulation d. is caused by umbilical cord compression

71.When should the nurse assess Mrs. Og's blood pressure?

a. During the Increment of a contractionb. Between contractions c. During the decrement of a contractiond. During the acme of a contraction

72.Which factor would be most helpful in assessing the adequacy of placentas perfusion in Mrs Og?

a. The duration and intensity of her contractionsb. Her ability to cope with the discomfort of laborc. The duration of the rest phases between contractions d. The effectiveness of her breathing techniques during a contraction

73. A nurse is demonstrating cord care to a mother of a neonate. Which actions would the nurse teach the mother to perform?

I. Keep the diaper below the cord II. Tug gently on the cord as it begins to dryIII. Only sponge bath the infant until the cord fails offIV. Apply antibiotic ointment to the cord twice daily

a. I, II b. I, III c. I, IV d. II, III

74. A nurse is caring for a 3-year-old with viral meningitis. Which signs and symptoms would the nurse expect to find during the initial assessment?

a. Bulging anterior fontanel, fever, nuchal rigidityb. Fever, nuchal rigidity, petechiae c. Hypothermia, photophobia, irritability d. Fever, nuchal rigidity, photophobia

75.The nurse teaches that the most

frequent side effect associated with the use of IUDs is:

a. ectopic pregnancy b. Expulsion of the IUD c. Rupture of the uterusd. Excessive menstrual flow

76.A client seeking advice about contraception asks the nurse about an IUD. The nurse explains that the IUD provides contraception by:

a. Blocking the cervical osb. Increasing the mobility of the uterus c. Preventing the sperm from reaching the vaginad. IUDs interfere with either fertilization or implantation. Promoting contraception

77. A diagnostic test used to evaluate the fertility is the post coital test. It is best timed:

a. 1 week after ovulation b. Immediately after menses c. Just before the next menstrual period d. With in 1 to 2 days of presumed ovulation

78. One of the most common causes of hypotonic uterine dystocia is:

a. Twin gestation b. Maternal anemia c. Pelvic contracture d. Pregnancy-induced hypertension

79. The safest position for the woman in labor when the nurse notes a prolapsed cord is:

a. Proneb. Fowlers c. Lithotomyd. Trendeirnburg

80. A birth hazard associated with breech delivery may be:

a. Abruptio placenta b. Cephalhematoma c. Pathologic jaundice d. Compression of the cord

81 .The nurse reaches the client that gonorrhea is highly infectious and:

a. is easily cured

Page 45: Document1

b. Occurs very rarelyc. Can produce sterilityd. Is limited to the external genitalia

82. When the client is diagnosed as having gonorrhea, the nurse should expect the physician to order:

a. Colistinb. Ceftriaxone c. Actinomycin d. Chloramphenicol

83. The nurse understands that the organism that causes a trichomonal infection is a:

a. Yeast b. Fungus c. Protozoand. Spirochete

84. Tile oral drug that is most likely to be prescribed for treatment of Trichomonas vaginalis is:

a. Penicillinb. Gentian violetc. Nystatin (Mycostatin)d. Metronidazole (Flagyl)

85. Acute salpingitis is most commonly the result of:

a. Syphilisb. Abortionc. Gonorrhead. Dydatidiform mole

86. Syphilis is not considered contagious in the:

a. Tertiary stage b. Primary stage c. incubation staged. Secondary stage

87. When teaching a client about the drug therapy for gonorrhea, the nurse should state that it:

a. Cures the Infection b. Prevents complications c. Controls its transmission d. Reverses pathologic changes

88. With cancer of the prostate, it is possible to follow the course of the disease by , monitoring the serum level

of:

a. Creatinine b. Blood urea nitrogen c. Non protein nitrogend. Prostate specific antigen

89. A client is diagnosed with herpes genitalis. To prevent cross contammiation, the nurse should:

a. Institute droplet precautionsb. Arrange transfer to a private room c. Wear a gown and gloves when giving direct care d. Close the door and wear a mask when in the room

90. A nurse should be aware that benign prostatic hypertrophy:

a. is a congenital abnormalityb. Usually becomes malignantc. Predispose to hydronephrosisd. Causes an eievated acid phosphatase

91. A 4-year-old has a seizure disorder and has been taking phenytoin(Dilantin) for 3 years. An important nursing measure for the child would be to:

a. Offer the urinal frequentlyb. Check for pupilary reaction c. Observe for flushing of the faced. Administer scrupulous oral hygiene

92. In terms of preventive teaching for the parents of a 1-year-old, the nurse would speak to them about:

a. Accidents b. Toilet training c. Adequate nutrition d. Sexual development

93. The best choice for between meal nourishment for a preschool-age child with a urinary infection would be:

a. Skim milk b. Fresh fruit c. Hard candy d. Creamed soup

94.When performing a physical assessment of a newborn with Down Syndrome, thenurse should carefully evaluate the infant's:

a. Heart sounds b. Anterior fontanel c. Pupillary reaction d. Lower extremities

95. If monocular strabismus in children is not corrected early enough:

a. Dyslexia will develop b. Peripheral vision will disappear c. Amblyopia develops in the weak eye d. Vision in both eyes will be diminished

96. Chickenpox can sometimes be fatal to children who are receiving:

a. Insulin b. Steroids c. Antibiotics d. Anticonvulsant

97. A viral Infection characterized by a red blotchy rash and Koplik's spots in the mouth is:

a. Mumps b. Rubellac. Rubeola d. Chickenpox

98.The major influence of eating habits of the early school-aged child is:

a. Availabilitv of food selections b. Smell and appearance of food c. Example of parents at meal time d. Food preferences of the peer group

99. Nursing care for an infant after the surgical repair of a cleft lip should include:

a. Keeping the baby NPOb. Keeping the infant from cryingc. Placing the infant in a semi-sitting positiond. Spoon feeding for the first 2 days after surgery

100. When teaching the parents of an infant diagnosed with PKU, the nurse should plan to include the fact that:

a. Mental retardation occurs if PKU is untreated b. Treatment for PKU includes life long medications c. PKU is transmitted by an autosomal

Page 46: Document1

dominant gene d. The infant is tested for PKU immediately after delivery

Page 47: Document1

Situation 1 - Jimmy developed this goal for hospitalization. "To get a handle on my nervousness." The nurse is going to collaborate with him to reach his goal. Jimmy was admitted to the hospital because he called his therapist that he planned to asphyxiate himself with exhaust from his car but frightened instead. He realized he needed help.

1. The nurse recognized that Jimmy had conceptualized his problem and the next priority goal in the care plan is:

a. help the client find meaning in his experienceb. help the client to plan alternativesc. help the client cope with present problemd. help the client to communicate

2. The nurse is guided that Jimmy is aware of his concerns of the "here and now" when he crossed out which item from this "list of what to know"

a. anxiety laden unconscious conflictsb. subjective idea of the range of mild to severe anxietyc. early signs of anxietyd. physiological indices of anxiety

3. While Jimmy was discussing the signs and symptoms of anxiety with his nurse, he recognized that complete disruption of the ability to perceive occurs in:

a. panic state of anxietyb. severe anxietyc. moderate anxietyd. mild anxiety

4. Jimmy initiates independence and takes an active part in his self care with the following EXCEPT:

a. agreeing to contact the staff when he is anxiousb. becoming aware of the conscious feelingc. assessing need for medication and medicating himselfd. writing out a list of behaviors that he identifies as anxious

5. The nurse notes effectiveness of Interventions in using subjective and objective data in the:

a. initial plans or orderb. databasec. problem listd. progress notes

Situation 2 - A research study was under taken in order to identify and analyze a disabled boy's coping reaction pattern during stress.

6. This study which is a depth study of one boy is a:

a. case studyb. longitudinal studyc. cross-sectional studyd. evaluative study

7. The process recording was the principal tool for data collection. Which of the following is NOT a part of a process recording?

a. Non verbal narrative accountb. Audio and interpretationc. Audio-visual recordingd. Verbal narrative account

8. Which of these does NOT happen in a descriptive study?

a. Exploration of relationship between two or more phenomenab. Exploration of relationships between two or more phenomenac. Manipulation of phenomenon in real life contextd. Manipulation of a variable

9. The investigator also provided the nursing care of the subject. The investigator is referred to as a/an.

a. Participant-observerb. Observer researcherc. Caregiverd. Advocate

10. To ensure reliability of the study, the investigator analysis and interpretations were:

a. subjected to statistical treatmentb. correlated with a list coping behaviorsc. subjected to an inter-observe agreementd scored and compared standard criteria

Situation 3 - During the morning endorsement, the' outgoing nurse informed the nursing staff that Regina, 5 years old, was given Flurazepam (Dalmane) 15 mg at 10:00pm because she had trouble going to sleep. Before approaching Regina, the nurse read the observation of the night nurse.

11. Which of the following approaches of the nurse validates the data gathered?

a. "I learned that you were up till ten last night, tell me what happened before you were finally able to sleep and how was your sleep?"b. "Hmm...You look like you had a very sound sleep. That pill you were given last night is effective isn't it?"c. "Regina, did you sleep we!!?" d. "Regina, how are you?"

12. Regina is a high school teacher. Which of these information LE^ST communicate attention and care for her needs for information about her medicine?

a. Guided by a medication teaching plan go over with her the purpose, indications and special instructions, about the medication and provide her a checklist b. Provide a drug literature c. Have an informal conversation about the medication and its effectsd. Ask her what time she would like to watch the informative video about the medication

13. The nurse engages Regina in the process of mutual inquiry to provide an opportunity for Regina to

a. face emerging problems realisticallyb. conceptualize her problemc. cope with her present problemd. perceive her participation in an experience

14. Which of these responses indicate that Regina needs further discussion regarding special instructions?

a. "I have to take this medicine judiciously."b. "I know 1 will stop taking the medicine when there is an advice form the doctor for me to discontinue."c. "I will inform you and the doctor any untoward reactions I have."

Page 48: Document1

d. "I like taking this sleeping pill. It solves my problem of insomnia. I wish I can take it for life."

15. Regina commits to herself that she understood and will observe all the medicine precautions by;

a. affixing her signature to the teaching plan that she has understood the nurseb. committing what she learned to her memoryc. verbally agreeing with the nursed. relying on her husband to remember the precautions

Situation 4 - The nurse-patient relationship is a modality through which the nurse meets the client's needs.

16. The nurse's most unique tool in working with the emotionally ill client is his/her:

a. theoretical knowledgeb. personality make upc. emotional reactionsd. communication skills

17. The psychiatric nurse who is alert to both the physical and emotional needs of clients is working from the philosophical framework that states:

a. All behavior is meaningful, communicating a message or a needb. Human beings are systems of interdependent and interrelated parts c. Each individual has the potential for growth and change in the direction of positive mental healthd. There is a basic similarity among all human beings

18. One way to increase objectivity in dealing with one’s fears and anxieties is through the process of:

a. observationb. interventionc. validationd. collaboration

19. All of the following response are non therapeutic. Which is the MOST direct violation of the concept, congruence of behavior?

a. Responding in a punitive manner to the

clientb. Rejecting the client as a unique human beingc. Tolerating all behavior in the client d. Communicating ambivalent messages to the client

20. The rnentally ill person responds positively to the nurse who is warm and caring. This demonstration of the nurse’s role as:

a. counselorb. mother surrogatec. therapistd. socializing agent

Situation 5 - The nurse engages the client in a. nurse-patient interaction.

21. The best time to inform the client about terminating the nurse-patient relationship is

a. when the client asks, how long one relationship would beb. during the working phasec. towards the end of the relationshipd. at the start of the relationship

22. The client says, "I want to tell you something but can you promise that you will keep this, a secret?" A therapeutic response of the nurse is:

a. "Yes, our interaction is confidential provided the information you tell me is not detrimental to your safety."b. "Of course yes, this is just between you and me. Promise!"c. "Yes, it is my principle to uphold my client's rights." d. "Yes, you have the right to invoke confidentiality of our interaction."

23. When the nurse respects the client's self-disclosure, this is a gauge for the nurse's:

a. trustworthinessb. loyaltyc. integrityd. professionalism

24. Rapport has been established in the nurse-client interaction time. I am committed to have this time available for us while you are at the hospital and ends after your discharge."

a. "The best time to talk is during the nurse-client interaction time. I am committed to have this time available for us while you are at the hospital and ends after your discharge." b. "Yes, if you keep it confidential, this is part of privileged communication." c. "I am committed for your care."d. "I am sorry, though I would want to, it is against hospital policy."

25. The client has not been visited by relatives for months. He gives a, telephone number and requests the nurse to call. An appropriate action of the nurse would be:

a. Inform the attending psychiatric about the request of the clientb. Assist the client to bring his concern to the attention of the social workerc. "Here (gives her mobile phone). You may call this number now."d. Ask the client what is the purpose of contacting his relatives

Situation 6 - Camila, 25 years old, was reported to be gradually withdrawing and isolating herself from friends and family members. She became neglectful of her personal hygiene. She was observed to be talking irrelevantly and incoherently. She was diagnosed as schizophrenia

26. The past history of Camila would most probably reveal that her premorbid personality is:

a. schizoidb. extrovertc. ambivertd. cycloid

27. Camila refuses to relate with others because she:

a. is irritableb. feels superior of othersc. anticipates rejectiond. is depressed

28. Which of the following disturbances in interpersonal relationships MOST often predispose, to the development of schizophrenia?

a. Lack of participation in peer groupsb. Faulty family atmosphere and

Page 49: Document1

interactionc. Extreme rebellion towards authority figuresd. Solo parenting

29. Camila's indifference toward the environment is a compensatory behavior to overcome:

a. Guilt feelingsb. Ambivalencec. Narcissistic behaviord. Insecurity feelings

30. Schizophrenia is a/an:

a. anxiety disorderb. neurosisc. psychosisd. personality/disorder

Situation 7 - Salome, 80 year old widow, has been observed to be irritable, demanding and speaking louder than usual. She would prefer to be alone and take her meals by herself, minimized receiving visitors al home and no longer bothers to answer telephone calls because of deterioration of her hearing. 'She was brought by her daughter to, the Geriatic clinic for assessment and treatment.

31. The nurse counsels Salome's daughter that Salome's becoming very loud and tendency to become aggressive is a/an:

a. beginning indifference to the world around her b. attempt to maintain authoritative rolec. overcompensation for hearing lossd. behavior indicative of unresolved repressed conflict of the part

32. A nursing diagnosis for Salome is:

a. sensory deprivationb. social isolationc. cognitive impairmentd. ego despair

33. The nurse will assist Salome and her daughter to plan a goal which is:

a. adjust to the loss of sensory and .perceptual functionb. participate in conversation and other social situationsc. accept the steady loss of hearing that occurs with aging

d. increase her self-esteem to maintain her authoritative role

34. The daughter understood, the following ways to assist Salome meet her needs and avoiding which of the following:

a. Using short simple sentencesb. Speaking distinctly and slowlyc. Speaking at eye level and having the client's attentiond. Allowing her to take her meals alone

35. Salome was fitted a hearing aid. She understood the proper .use and wear of this device when she ways that the battery should be functional, the device is turned on and adjusted to a:

a. therapeutic levelb. comfortable levelc. prescribed leveld. audible level

Situation 8 - For more than a month now, Cecilia is persistently feeling restless, worried and feeling as if something dreadful is going to happen. She fears being alone in places and situations where she thinks that no one might come to rescue her just in case something happens to her.

36. Cecilia is demonstrating:

a. acrophobiab. claustrophobiac. agoraphobiad. xenophobia

37. Cecilia's problem is that she always sees and thinks negative hence she is always fearful Phobia is a symptom described as:

a. organicb. psychosomaticc. psychoticd. neurotic

38. Cecilia has a lot of irrational thoughts: The goal of therapy is to modify her:

a. communicationb. cognitionc. observationd. perception

39. Cognitive therapy is indicated for Cecilia when she is already able to handle anxiety reactions. Which of the following should the nurse implement?

a. assist her in recognizing irrational beliefs, and thoughtsb. help find meaning in her behaviorc. provide positive reinforcement for acceptable behaviord. administer anxiolytic drug

40. After discharge, which of these behaviors indicate a positive result of being able to overcome her phobia?

a. she read a book in the public libraryb. she drives alone along the long expressway c. she watches television with the family in the recreation roomd. she joint an art therapy group

Situation 9 - it is the first day of clinical experience of nursing students at the Psychiatry Ward- During the orientation, the nurse emphasizes that the team members including nursing students are legally responsible to safeguard patient's records from loss or destruction or from people not authorized to bead it.

41. It is unethical to tell one's friends and family member’s data bout patients because doing so is violation of patients’ rights to:

a. Informed consentb. Confidentialityc. Least restrictive environmentd. Civil liberty

42. The nurse must see to it that the written consent of mentally ill patients must be taken from:

a. Doctorb. Social workerc. Parents or legal guardiand. Law enforcement authorities

43. In an extreme situation and when no other resident or intern is available, should a nurse receive, telephone orders, the order has to be correctly written and signed by the physician within.

a. 24 hoursb. 36 hours

Page 50: Document1

c. 48 hoursd. 12 hours

44. The following are SOAP (Subjective - Objective - Analysis - Plan) statements on a problem: Anxiety about diagnosis. What is the objective data?

a. Relate patient's feelings to physician initiate and encourage her to verbalize her fears give emotional support by spending more time with patient, continue to make necessary explanations regarding diagnostic test.b. Has periods of crying, frequently verbalizes fear of what diagnostic tests will revealc. Anxiety due to the unknownd. "I’m so worried about what else they'll find wrong with me"

45. Nursing care plans provide very meaningful data for the patient profile and initial plan because the focus is on the:

a. Summary of chronological notations made by individuals health team members b. Identification of patient's responses to medical diagnosis and treatment c. Patient's responses to health: and illness as a total person in interaction with the environmentd. Step procedures for the management of common problems

Situation 10 - Marie is 5 ½ years old and described by the mother as bedwetting at night.

46. Which of the following is the MOST common physiological cause of night bedwetting?

a. deep sleep factorsb. abnormal bladder development or structure problemsc. infections familial and genetic factors

47. All of the following, EXCEPT one comprise the concepts of behavior therapy program:

a. reward and punishmentb. extinctionc. learningd. placebo as a form treatment

48. The help Marie who bed wets at night

practice acceptable and appropriate behavior, it is important for the parents to be consistent with the following approaches EXCEPT:

a discipline with a king attitudeb matter of fact in handling the behaviorc. sympathize for the childd. be lowing yet firm

49. A therapeutic verbal approach that communicates strong disapproval is:

a. You are supposed to get up and go in the toilet when you feel you have to go and did not. The next time you bed wet, I’ll tell your friends and hand your sheets out the window for them to see."b. "You are supposed to get up and go in the toilet when you feel you have to go and you did not. I expect you to from now on without fail." c. "If you bed wet, you will change your bed linen and wash the sheets." d. "If you don't make an effort to control your bedwetting, I'd be upset and disappointed."

50. During your conference, the parent inquires how to motivate Marie to be dry in the morning. Your response which is an immediate intervention would be:

a. Give a star each time she wakes up dry and every set of five stars, give a prize b. Tokens make her materialistic at an early age. Give praise and hugs occasionally c. What does you child want that you can give every time he/she wakes up dray in the morningd. Promise him/her a long awaited vacation after school is over.

Situation 11 - The nurse is often met with t-he following situations when clients become angry and hostile.

51. To maintain a therapeutic eye contact and body posture while interacting with angry and aggressive individual, the nurse should:

a. keep an eye contact while staring at the clientb. keep his/her hands behind his/her back or in one's pocketc. fold his/her arms across his/her chest c. keep an "open" posture, e.g. Hands by

sides but palms turned outwards

52. During the pre-interaction phase of the N-P relationship/the nurse recognizes this normal INITIAL reaction to an assaultive or potentially assaultive person.

a. To remain and cope with the incidentb. Display empathy towards the patient c. To call for help from the other members of the teamd. To stay and fight or run away

53. Which of the following is an accurate way of reporting and recording an incident?

a. "When asked about his relationship with his father, client became anxious." b. "When asked about his relationship with his father, client clenched his jaw/teeth made a fist and turned away from the nurse."c. "When asked about his relationship with his father, client was resistant to respond."

d. "When asked about his relationship with his father, his anger was suppressed."

54. To encourage thought. Which of the following approaches is NOT therapeutic?

a. "Why do you feel angry?"b. "When do you usually feel angry?"c. "How do you usually express anger?" d. "What situations provoke you to be angry?"

55. A patient grabs a chair and about to throw it. The nurse best responds saying.

a. "Stop! Put that chair down."b. "Don't be silly."c. "Stop, the security will be here in a minute."d. "Calm down."

Situation 12 - Nursing care for the elderly.

56. In planning care for a patient with Parkinson's disease, which of these nursing diagnoses should have priority?

a. potential for injuryb. altered nutritional statec. ineffective copingd. altered mood state

Page 51: Document1

57. A healthy adaptation to aging is primarily related to an individual.

a. Number of accomplishments b. Ability to avoid interpersonal conflictc. Physical health throughout lifed. Personality development in his life span

58. The frequent use of the older client's name by the nurse is MOST effective in alleviating which of the following responses to old age?

a. Loneliness b. Suspicion c. Griefd. Confusion

59. An elderly confused client gets out of bed at night to go to the bathroom and tries to go to another bed when she returns. The MOST appropriate action the nurse would take is to:

a. Assign client to a single roomb. Leave a light on all nightc. Remind client to call the nurse when she wants to get upd. put side rails on the bed

60. An elderly who has lots of regrets, unhappy and miserable1 is experiencing:

a. Crisisb. Despairc. Lossd. Ambivalence

Situation 13 - Graciela 1 ½ year old is admitted the hospital from the emergency room with a fracture of the left femur due to a Tall down a flight of stairs. Graciela is placed oh Bryant's traction.

61. While on Bryant's traction, which of these observations of Graciela and her traction apparatus would indicate a decrease in the effectiveness of her traction?

a. Graciela's buttocks are resting on the bed b. The traction weights are hanging 10 inches above the floor c. Graciela's legs are suspended at a 90 degree angle to her trunk d. The traction ropes move freely through the pulley

62. The nurse notes that the fall might also cause a possible head injury. She will be observed for signs of increased intracranial pressure which include:

a. Narrowing of the pulse pressureb. Vomitingc. Periorbital edema d. A positive Kernig's sign

63. Graciela is assessed to have no head injury. The Bryant's traction is removed. A plaster of Paris his spica is applied. Which of these finding as a concern of immediate attention that must be reported to the physician immediately?

a. Graciela is scratching the cast over her abdomenb. The toes of Graciela's left foot blanch when the nurse applies pressure on themc. Graciela's cast is still dampd. The nurse is unable to insert a finger under the edge of Graciela's cast on her left foot

64. Part of discharge plan is for the nurse to give instructions about the care of Graciela's cast to the mother. Which of these statements indicate that the mother understood an important aspect of case care?

a. I will use white shoe polish to keep the cast neatb. I will place plastic sheeting around the perineal area of the castc. I will use cool water to wash the castd. I will reinforce cracked areas on the cast with adhesive tape

65. The nurse counsels Graciela's mother ways to safeguard safety white providing opportunities of Graciela to develop a sense of:

a. Trustb. Initiativec. Industryd. Autonomy

Situation 14 - Jolina is an 18 year old beginning college student. Her mother observed that she is having problems relating with her friends. She is undecided about her future. She has lost insight, lost interest in anything and complained and complained of constant tiredness.

66. Jolina is out on antidepressant drugs. These drugs act on the brain chemistry, therefore they would be useful in which type of depression?

a. exogenous depressionb. neurotic depressionc. endogenous depressiond. psychotic depression

67. This is a tricyclic antidepressant drug:

a. Venlafaxine (Effexor)c. Setraline (Zoloft)b. Flouxetine (Prozac)d. Imipramine (Tofranil)

68. After one week of antidepressant medication, Jolina still manifests depression. The nurse evaluates this as;

a. Unusual because action of antidepressant drug is immediateb. Unexpected because therapeutic effectiveness takes within a few daysc. Expected because therapeutic effectiveness takes 2-4 weeksd. Ineffective result because perhaps the drug's dosage is inadequate

69. Jolina continues to verbalize feeling sad and hopeless. She is not mixing well with other clients. One of the nurse's important consideration for Jolina Initially is to:

a. Formulate a structured schedule so she is able to channel her energies externallyb. Let her alone until she feels like mingling with othersc. Encourage her to join socialization hour so she will start to relate with othersd. Encourage her to join group therapy with other patients

70. During the predischarge conference, the nurse suggests vocational guidance because it should help Jolina to:

a. Find a good jobb. Make some decision about her futurec. Realistically assess her assets and limitationsd Solve her own problems

Situation 15 - Group Approach" in Nursing.

71. Membership dropout generally occurs in group therapy after a member:

Page 52: Document1

a. Accomplishes his goal in joining the groupb. Discovers that his feelings are shared by the group membersc. Experiences feelings of frustration in the groupd. Discusses personal concerns with group members

72. Which of the following questions illustrates the group role of encourager?

a. What were you saying?b. Who wants to respond next?c. Where do you go from here?d. Why haven't we heard from you?

73. The goal of remotivation therapy is to facilitate:

a. Insightb. Productivityc. Socializationd. Intimacy

74. The treatment of the family as a unit is based on the belief that the family:

a. is a social system and all the members are interrelated components of that systemb. as a unit of society needs the opportunity to change its own destinyc. who has therapy together will tend to remain togetherd. is "contaminated" by the presence of deviant member and all members need treatment

75. The working phase in therapy group is usually characterized by which of the following?

a. Cautionb. Cohesivenessc. Confusiond. Competition

Situation 16 - The mental health - psychiatric nurse functions in a variety of setting with different types of clients.

76. Poverty as reflected in prevalence of communicable diseases, malnutrition and social ills such as street children, homeless and prostitution is a predisposing factor to mental illness. A community approach to cope with this

problem is for the nurse to support:

a. aggressive family planning methods b. provision of social welfare benefits for the poorc. social actiond. free clinics and more hospitals

77. The MOST cost effective way to meet the mental health needs of the public is through programs with a priority goal of:

a. treatmentb. preventionc. rehabilitationd. research

78. Lorelle upon discharge was referred to a volunteer group where she has learned to read patterns, cut out fabric and use a sewing machine to make simple outfits that will help her earn in the future. What type of activity therapy is this?

a. Recreational therapyb. Art therapyc. Vocational therapyd. Educational therapy

79. In a residential treatment home for adolescent girl's the clients were becoming increasingly tense and upset because of shortening of their recreation time. To die escalate possible anger and aggression among the clients it is BEST to play:

a. religious music b. relaxation musicc. dance musicd. rock music

80. The parents of special children who are behaviorally disturbed need mental health education. Which of these topics would the school nurse consider as priority for their parents’ class?

a. Drug educationb. Child abusec. Effective parentingd. Sex education

Situation 17 - Nurse's in all practice areas are likely to come in contact with clients suffering from acute or chronic drug abuse.

81. The psychodynamic therapy of

substance abuse is based upon the premise that drug abuse is:

a. a common problem brought about by socioeconomic deprivationb. caused by multiplicity of factorsc. predisposed by an inability to develop appropriate psychological resources to manage developmental stresses d. due to biochemical factors

82. Being in contact with reality and the environment is a function of the:

a. conscienceb. egoc. idd. super ego

83. Substance abuse is different from substance dependence is than, substance dependence:

a. includes characteristics of adverse consequences and repeated useb. requires long term treatment in a hospital based programc. produces less severe symptoms than that of abused. includes characteristics of tolerance and withdrawal

84. During the detoxification stage, it is a priority for the nurse to:

a. teach skills to recognize and respond to health threatening situations b. increase the client's awareness of unsatisfactory protective behaviors c. implement behavior modificationd. promote homeostasis and minimize the client’s withdrawal symptoms

85. Commonly known as "shabu" is:

a. Cannabis Sativab. Lysergic add diethylamidec. Methylenedioxy, methamphetamined. Methamphetamine hydrochloride

Situation 18 - It is common that client ask the nurse personal questions.

86. Anticipation of personal questions is given adequate attention during which phase of the nurse patient relationship?

a. Orientation phaseb. Working phase

Page 53: Document1

c. Pre-interaction phased. Termination phase

87. The client asks for the nurse's telephone number, which of these responses is NOT appropriate?

a. "it is confidential I just don't give it to anyone."b. "What would you do with my number if I give it to you?"c. "If I say. No to your request, what are your thoughts about it?”d. "Are you asking for an official number of the hospital/clinic for your reference?"

88. When the client asks about the family of the nurse the MOST appropriate response is:

a. Avoid the situation and redirect the client's attentionb. Give a brief and simple response and focus on the clientc. "Why don't we talk about your family instead?"d. Introduce another topic like the client's interests

89. When the nurse is asked a personal question, which of these reactions indicate a need her to introspect?

a. The client is simply curiousb. His/her right to privacy is being intrudedc. The client knows no other way to begin a conversationd. Some patients are like children in seeking recognition from the nurse

90. It is 10 o'clock of your watch. The client asks, "What time is it?" The nurse's appropriate response is:

a. "Are you bored?"b. "It is 10 o’clock."c. "Why do you ask?"d. "Guess, what time is it?"

Situation 19 - Ricky is a 12 year old-boy with Down’s syndrome. He stands 5' ½" and weight 100 lbs. He is slim and walks sluggishly with a limp. He wears a neck brace as support for his neck. X - ray of cervical spine showed "subluxation of CI in relation to C2 with cord compression." He attends a school for special education.

91. The classroom teacher consults the

school for guidance on how to take care of Ricky while inside the, classroom. The nurse considers as priority, Ricky's:

a. Physiological needsb. Need for self-esteem c. Needs for safety and Securityd. Needs for belonging

92. Ricky's mother visited the school nurse. She asked, " What should I do when Ricky fond his genitalia?" Appropriate response of the nurse is for the mother to:

a. Divert Ricky's attention and engage him in satisfying activitiesb. Tell Ricky that it is wrong to keep fondling his genitalia c. Ignore Ricky's behavior because he will outgrow it laterd. Engage him in computer TV games that engage his hands

93. The nurse has one on one health education sessions with Ricky's mother. The mother understood that for her son to learn to cope and be independent, she should constantly provide activities for Ricky to be able to:

a. socialize with peopleb. eventually go to school alonec. select and prepare his own foodd. do activities of daily living

94. All of the following activities are appropriate for Ricky EXCEPT:

a. Working with clayb. Competitive sportsc. Preparing and cooking simple menud. Card and table games

95. Ricky's IQ falls within the range of 50-55. He can be expected to:

a. Profit from vocational training with moderate supervisionb. Live successfully in the communityc. Perform simple tasks in closely supervised settingsd. Acquire academic skills of 6th grade; level

Situation 20 - The abuse of dangerous drug is a serious public health concern that nurses need to address,

96. The nurse should recognize that the unit primarily responsible for education and awareness of the members of the family on the ill effects of dangerous drugs is the:

a. law enforcement agenciesb. schoolc. churchd. family

97. A drug dependent utilizes this defense mechanism and enables him to forget shame and pain.

a. repressionb. rationalizationc. projectiond. sublimation

98. This drug produces mirthfulness, fantasies, flight of ideas, loss of train of thought, distortion of size, distance and time, and "bloodshot eyes", due to dilated pupils.

a. Opiatesb. LSDc. Marijuanad. Heroin

99. The nurse evaluates that-.her health teaching to a group of high school boys is effective if these students recognize which of the following dangers of inhalant abuse.

a. Sudden death from cardiac or respiratory depression b. Danger of acquiring hepatitis or AIDSc. Experience of "blackout"d. Psychological dependence after prolonged use

100. The mother of a drug dependent would never consider referring her son to a drug rehabilitation agency because she fears her son might just becomes worse while relating with other drugs users. The mother's behavior can be described as:

a. Unhelpfulb. Codependent c. Caretakingd. Supportive

Page 54: Document1

ANSWER KEY:1. C2. C3. B4. A5. D6. A7. C8. D9. D10. A11. A12. D13. D14. D15. A16. D17. C18. B19. 20. 21. 22. 23. 24. 25. A26. A27. A28. B29. C30. C31. A32. A33. A

34. D35. D36. C37. D38. B39. A40. A41. B42. C43. A44. B45. C46. 47. 48. 49. 50. 51. D52. B53. B54. A55. A56. A57. C58. D59. A60. B61. A62. B63. D64. D65. D66. B67. D

68. C69. C70. C71. C72. B73. B74. A75. B76. B77. B78. C79. B80. C81. B82. B83. D84. D85. D86. B87. A88. B89. D90. B91. C92. 93. D94. B95. C96. D97. A98. B99. A100. A